Download ID_4096_Nephrology (Current practice i_English_sem_12

Survey
yes no Was this document useful for you?
   Thank you for your participation!

* Your assessment is very important for improving the workof artificial intelligence, which forms the content of this project

Document related concepts

Prenatal testing wikipedia , lookup

List of medical mnemonics wikipedia , lookup

Transcript
Назва наукового напрямку (модуля):
Семестр: 12
Nephrology (Current practice in internal medicine)
Опис:
situational tasks
Перелік питань:
1.
A.
B.
C.
D.
E. *
2.
A.
B.
C.
D.
E. *
3.
A.
B.
C. *
D.
E.
A 64-year-old man comes to the office complaining of left flank pain and gross hematuria. The pain
is non-radiating, dull in nature, and not associated with any nausea or vomiting. The hematuria is
painless, intermittent, and not associated with any fevers or chills. The patient has not had any
medical care in over 20 years. His vital signs and physical examination are unremarkable. A
complete blood count and biochemical profile are normal. A urinalysis shows full field red blood
cells. A CT scan of abdomen and pelvis reveals a contrast enhancing 5cm mass in the lower pole of
the left kidney. A chest x-ray shows no signs of metastasis. The most appropriate intervention at this
order
CT guided percutaneous needle biopsy
time isa to
order an MRI with gadolinium
schedule chemotherapy
schedule radiation therapy
send him to a urologist for a radical nephrectomy
A 26-year-old men is brought to the office because of a "red rash" that she noticed today. She says
that 3 days ago he had a cough, runny nose, and fever that responded to ibuprofen. In the office, his
temperature is 37 oC (98.6 F) and he has a normal physical examination with the exception of an
erythematous, blanching macular rash on his legs. You diagnose him with a viral exanthem and
advise to drink liquids and to use ibuprofen as needed for fever. One week later, the men comes back
to the office and reports that the rash has "changed", he has developed colicky abdominal pain
several times per day, and he is complaining of left knee pain. In the office, his temperature is 37.2 C
(99 F), blood pressure is 100/65 mm Hg, pulse is 100/min, and respiratory rate is 15/min. A physical
examination reveals a well-appearing patient with palpable purpura of both lower extremities, normal
neck examination, clear lungs, and a soft, non-tender abdomen. His left knee is painful on flexion,
but it is not erythematous or warm, and there does not seem to be an effusion. His gait is normal. The
most appropriate study at this time is
arthrocentesis
colonoscopy
cultures of blood, urine, and cerebrospinal fluid
echocardiography
urinalysis
A 23-year-old woman comes to the clinic for a pre-employment examination. She recently moved to
the area from out of state and got a job at a local small business. Her past medical history is
significant only for diabetes mellitus type I, which she has had since age 13. Her only medication is
insulin, which is infused via an insulin pump. She denies smoking or using illicit drugs. She admits to
social alcohol consumption less than once a week and says she runs 2 miles daily. She is not sexually
active. You perform a full physical examination. Her temperature is 37.1 (98.8 F), blood pressure is
136/89 mm Hg, pulse is 54/min, and respirations are 12/min. Her skin is warm and dry.
Cardiovascular examination reveals a normal S1, S2 with no murmurs appreciated. Respirations are
equal bilaterally without any abnormal breath sounds. Extremities show no clubbing, cyanosis, or
edema. Strength is equal bilaterally and sensation is full throughout. The patient exhibits normal
reflexes. She returns to the clinic several more times and her blood pressure remains elevated. The
most appropriate initial pharmacotherapy for this patient is
amlodipine
atenolol
enalapril
furosemide
hydrochlorothiazide
4.
A. *
B.
C.
D.
E.
5.
A.
B.
C.
D. *
E.
6.
A.
B.
C. *
D.
E.
A 57-year-old woman with diabetes and hypertension comes to the office because of a 5-day history
of weakness and lethargy. The symptoms developed slowly but have worsened over the last 2 days.
She denies any chest pain, dyspnea, fever, cough, abdominal pain, or dysuria, but has noticed a
decrease in her urine output. One week prior to admission she underwent a CT scan of the abdomen
with intravenous contrast for routine follow up of an abdominal aortic aneurysm, which is stable at 4
cm. She regularly takes hydrochlorothiazide, glyburide, captopril, and aspirin. Her temperature is
37.0 C (98.6 F), blood pressure is 165/94 mm Hg, pulse is 92/min, and respirations are 14/min.
Physical examination shows 2+ pitting edema in the lower extremities and 1+ peripheral pulses.
Urinalysis shows trace protein, 1-3 white blood cells, tubular casts and no red casts. This condition
most likely would have been prevented by
administering acetylcysteine prior to the contrast for the CT scan
holding her captopril 2 days prior to the CT scan
giving her intravenous ampicillin during the CT scan
giving her intravenous furosemide 1 hour prior to the CT scan
premedicating her with nifedipine 2 days prior to the CT scan
A 42-year-old woman comes into the clinic complaining of intermittent easy bruising around her
eyes and chest, especially after surfing and boogie boarding with her son. She recently suffered a
syncopal episode and a work up, which included a stress echocardiogram, revealed a hypertrophic
heart with a speckled pattern. When she was discharged from the hospital, she was given a diagnosis
of congestive heart failure with a restrictive pattern. An endomyocardial biopsy is scheduled for next
week. She denies any significant family history and has been healthy with the exception of easy
bruising, occasionally with vomiting. Routine urinalysis from her recent admission revealed
proteinuria. Upon further questioning, she has suffered carpal tunnel syndrome bilaterally and
occasional numbness and tingling of her toes. The most useful study to diagnose this patient's
complete
count
condition blood
is
liver function test
morning cortisol level
protein immunoelectrophoresis
thyroid function tests
A 55-year-old man comes to the emergency department with pain on urination, fever and chills. He
also complains of perineal and suprapubic tenderness as well as dysuria and hesitancy. His allergies
include codeine, sulfonamides, and quinidine. Temperature is 38.5 C (101.3 F), blood pressure is
132/90 mm Hg, pulse is 88/min, and respirations are 18/min. Abdominal examination is remarkable
for suprapubic tenderness. Digital rectal examination demonstrates a swollen, boggy, and exquisitely
painful prostate gland. Laboratory studies show a leukocyte count of 11,500/mm3, creatinine of 0.9
mg/dL, and blood urea nitrogen of 16 mg/dL. A urinalysis shows too numerous to count white blood
cells and Gram-negative rods. The most appropriate treatment for this patient is
amoxicillin/clavulanate 875 mg by mouth twice daily for 14 days
ceftriaxone 1 gram intravenously daily for 5 days
ciprofloxacin 500 mg by mouth twice daily for 14 days
clindamycin 300 mg 4 times daily for 10 days
trimethoprim-sulfamethoxazole 1 double strength tablet twice daily for 14 days
7.
A. *
B.
C.
D.
E.
8.
A. *
B.
C.
D.
E.
9.
A. *
B.
C.
D.
E.
10.
A. *
B.
C.
D.
E.
A 38-year-old man is admitted to the hospital for acute deterioration in renal function. He was seen in
your office 2 days prior for some mild upper respiratory complaints, including a sore throat, cough,
and fever. He was prescribed cephalexin and sent home. Today, his laboratory data returned and
shows a blood urea nitrogen level of 67 mg/dL and a creatinine level of 2.1 mg/dL. You called him
and told him to meet you at the hospital for further evaluation. On admission his BUN is now 109
mg/dL and his creatinine is 4.2 mg/dL. The appropriate tests are ordered and an electrocardiogram
shows QRS complex widening and tall, peaked T waves. His temperature is 38.3 C (101.0 F). He has
an erythematous oropharynx with some mild tonsillar exudate. His lungs are clear. It is observed that
he has urinated only 5-10 cc in the past 2 hours since his hospitalization. A urinalysis shows red cell
casts and dysmorphic red blood cells. The most appropriate next step is to
administer high-dose methyl-prednisolone, intravenously
administer low-dose methyl-prednisolone, intravenously
administer penicillin, intravenously
prescribe high-dose cyclophosphamide, orally
prescribe penicillin, orally
A 40-year-old man complained of headache in occipital area. On physical examination, the skin was
pale; there was face and hand edema, blood pressure of 170/130 mm Hg. On EchoCG, there was
concentric hypertrophy of the left ventricle. Ultrasound examination of the kidneys revealed thinned
cortical layer. Urinalysis showed proteinuria of 3.5 g/day. What is the diagnosis
Chronic glomerulonephritis
Essential arterial hypertension
Chronic pyelonephritis
Polycystic disease of the kidneys.
Cushing’s disease
A 29-year-old woman is critically ill. The illness was manifested by high fever, chills, sweating,
aching pain in lumbar area, a discomfort in urination, and frequent voiding. Pasternatsky’s sigh is
positive in both sides. On lab exam, WBC of 20.000/mcL; on urinalysis protein of 0.6g/L,
leukocyturia, bacteriuria. Your preliminary diagnosis.
Acute pyelonephritis
Exacerbation pf chronic pyelonephritis
Acute glomerulonephritis
Acute cystitis
Nephrolithiasis
A 25-year-old woman complained of fatigue, hair loss, and brittle nails. On exam, pallor of skin, PR
of 94/min, BP of 110/70 mm Hg. On blood cell count, Hb of 90 g/L, RBC 3.5·1012/L, color index of
0.7, ESR of 20 mm/h. Serum iron level was 8.7 mcmol/L. What treatment would you initiate?
Ferrous sulfate orally
Iron dextrin injections
Vitamin B12 intramuscularly
Blood transfusion
Packed RBCs transfusion
11.
A.
B.
C.
D.
E. *
12.
A. *
B.
C.
D.
E.
13.
A.
B.
C.
D. *
E.
A 58-year-old alcoholic with hepatitis C cirrhosis is admitted to the hospital for management of his
ascites. He has been managed as an outpatient with diuretics and oral lactulose, but over the past few
weeks, he reports increasing abdominal girth, weight gain and lower extremity edema. He has been
noncompliant with his low-sodium diet. His medications include furosemide, spironolactone,
lactulose, ciprofloxacin, and thiamine. On physical examination, he appears grossly edematous and
appropriately responsive. His lungs are clear and his heart is without extra sounds or murmurs. His
abdomen is tense with a fluid wave and shifting dullness on percussion. He has numerous
non-blanching telangiectasias on his torso and abdomen. His testes are small for his age and there is
no asterixis. Admission laboratory studies show: Sodium 121 mEq/L, Potassium 4.3 mEq/L,
Bicarbonate 29 mEq/L, BUN 38 mg/dL, Creatinine 1.5 mg/dL. Urinalysis shows some granular casts
and a urinary sodium concentration of <10 mmol/L. The most appropriate therapy is to
administer hypertonic saline
administer sodium chloride tablets
increase the dose of furosemide
increase the dose of spironolactone
salt restrict
A 20- year-old patient with a history of preceding streptococcal infection complains of malaise,
headache, anorexia, subfebrile fever. On exam, mild generalized edema, BP of 150/90 mm Hg, HR of
100/min, RR of 20/min, the urinalysis showed increased protein, red cell casts and hyaline casts.
What is your diagnosis
Acute glomerulonephritis
Acute pyelonephritis
Rheumatic fever
Essential hypertension
Bacterial endocarditis
A 29-year-old woman, who has been undergoing treatment for hypertension for the past 2 years,
comes to the office because of chills and right-sided flank pain. She has had chronic "low back pain"
that you have been treating unsuccessfully with nonsteroidal anti-inflammatory drugs. She has never
had any diagnostic studies performed to evaluate her hypertension or back pain. She is estranged
from her family, but she knows that her mother and brother have been treated for hypertension
starting at age 25. She has not spoken to them in 10 years and so she is unaware of any other medical
conditions. Her temperature is 38.1 C (100.6 F), blood pressure is 130/90 mm Hg, and pulse is
65/min. On physical examination today there is marked right-sided flank tenderness. Cardiac
examination reveals a mid-systolic click. Urinalysis shows pyuria and white blood cell casts. You
prescribe a 14-day course of trimethoprim-sulfamethoxazole, schedule a renal ultrasound and a
follow-up visit. You tell her to call your office immediately if the symptoms worsen. On the return
visit, she says that she feels much better, but still has dull flank pain. Her temperature is 37.0 C (98.6
F). The ultrasound report is in the chart and states that there are 7 cysts in her right kidney and 5 cysts
in her left kidney. At this time the most correct statement about her condition is:
Her hypertension is unrelated to the kidney disease
Long-term treatment with trimethoprim-sulfamethoxazole will eradicate the cysts
She has a 20% chance of developing end-stage renal disease by age 70
She is at an increased risk for developing colonic diverticular disease with perforation
There is a 70% chance that she has an intracranial aneurysm and will suffer from a subarachnoid
hemorrhage
14.
A.
B.
C.
D. *
E.
15.
A.
B.
C.
D.
E. *
16.
A. *
B.
C.
D.
E.
17.
A. *
B.
C.
D.
E.
A 27-year-old woman comes to the office for a periodic health maintenance examination. She started
a new job at a prestigious law firm 3 months ago and has been working very long hours. She feels "a
bit forgetful" lately, but she attributes that to the "midnight coffee runs" at the office. Her last
menstrual period was 3 weeks ago. She tells you that for the past 6 months, starting 7-10 days before
she is scheduled to get her period, she has been noticing some pinkish-red blood on the toilet paper
after she goes to the bathroom. It continues until she gets her period on the regularly scheduled day.
It has been somewhat annoying because it seems like she is bleeding for more than half of the month.
She is sexually active and she uses condoms as contraception. Her Pap smears over the past 10 years
have been normal. Her pelvic examination and physical examination are unremarkable. You perform
a Pap smear and send it for pathologic examination. A urine pregnancy test is negative. You notice
that she is putting lotion on her arms and hands as you are leaving the room. The most appropriate
begin
a therapeutic
trial of leuprolide acetate, parenterally
next step
is to
begin a therapeutic trial of naproxen, orally
determine thyroid stimulating hormone concentration
refer her for cervical dilatation and curettage of the entire uterine cavity
schedule a diagnostic laparoscopy
A 30-year-old woman comes to the clinic complaining of loss of control of urination and "dribbling"
of urine. She has had recurrent urinary tract infections over the past 11 months, which were treated
with antibiotics. She has been following safe sex practices and denies any history of sexually
transmitted diseases, but complains of moderate pain during intercourse. Physical examination and
pelvic examination are normal. The most appropriate next investigation in this patient is
CT scan of the abdomen and pelvis
laparoscopy
pelvic sonography
renal sonography
voiding cystourethrogram
A 78-year-old woman is admitted to the hospital because of fever, flank pain, and mental
obtundation. On arrival to the hospital, she was minimally responsive and was found to have a white
blood cell count of 43,000/mm3 with a profound left shift. Urinalysis revealed packed white cells. A
renal ultrasound demonstrated a left hydronephrosis and hydroureter. A CT scan confirmed the
presence of an obstructing stone. Her vital signs on admission showed a temperature of 39.5 C (103.1
F), blood pressure of 80/40 mm Hg, pulse of 112/min, and respirations of 18/min. Intravenous
pressors were initiated. Three sets of blood cultures came back positive for Gram-negative rods
within 2 hours. The most appropriate next step in management is to
insert a nephrostomy tube, percutaneously
administer antibiotics, intravenously, and observe
perform extracorporeal shock wave lithotripsy (ESWL)
schedule cystoscopy and ureteral stent placement
schedule a nephrectomy
A 37-year-old comes to the clinic for a required pre-employment physical examination. She has no
past medical history and has no complaints. Her temperature is 37.0 C (98.6 F), blood pressure is
110/60 mm Hg, pulse is 63/min, and respirations are 14/min. She is currently menstruating. A urine
culture, which is required by her new job, reveals greater than 100,000 E. Coli colony-forming units.
The most appropriate next step in management of this patient's urine culture findings is
no further management is indicated
prescribe ciprofloxacin, orally, for 14 days
prescribe trimethoprim-sulfamethoxazole, orally, for 3 days
repeat urine culture in 2 weeks
send her for urological evaluation
18.
A.
B.
C.
D.
E. *
19.
A.
B.
C.
D. *
E.
20.
A.
B.
C.
D. *
E.
21.
A. *
B.
C.
A 58-year-old woman comes to the office for a periodic health maintenance examination. She has no
complaints and is generally in good health. She takes no medications and does not drink or smoke.
You notice in her chart that her last menstrual period was two years ago, and at that time, she was not
interested in discussing hormone replacement therapy. Now she says that she has noticed that a few
of her friends have been "shrinking" and she is ready to take something "for osteoporosis." She has
been doing some research on the Internet and read that women with thromboembolic disease should
not take estrogen. She vaguely remembers having a few "blood clots" many years ago, before you
were her doctor, and so she wants to try "one of the newer drugs." You are not sure if she is
"remembering" this correctly, so you try to explain the benefits of estrogen replacement anyway, but
she still is not interested. Dual energy absorptiometry (DEXA) shows a bone mineral density that is
more than 2.5 standard deviations below the mean. Alendronate is prescribed. The patient should be
avoid
weight-bearing
exercise
advised
to
continue taking alendronate, even if she experiences some mild chest pain and gastrointestinal
symptoms, and return to the office as needed
return to the office in one week for a complete blood count (CBC)
take the alendronate, along with calcium, after breakfast
take the alendronate first thing in the morning, on an empty stomach, with a full glass of water, and
remain upright for at least 30 minutes
A 35-year-old woman is in the hospital for a flare of nephritis related to systemic lupus
erythematosus (SLE). On rounds in the morning, she complains of right hip pain. She states that for
the last several weeks, she has had a deep aching in the hip and now it is getting much worse since
she was in the hospital. It hurts her both at rest and with motion. She denies any history of trauma,
and has not started any new activities. There have been no fevers. Her only outpatient medication is
prednisone 10 mg daily, and she takes ibuprofen for pain relief, which has helped minimally.
However, now she is on a higher dose of intravenous steroid. Her temperature is 37.2 C (99 F), blood
pressure is 132/82 mm Hg, and pulse is 72/min. There is no pain on palpation over the hip but pain is
present with range of motion. Laboratory studies show a leukocyte count 8,100mm3 and a hematocrit
34%. A plain x-ray of the pelvis and hip is normal. The next most appropriate step is to
assure her that her pain will resolve spontaneously
continue the patient on oral ibuprofen and follow her symptoms as an outpatient in 1 month
increase the duration of her intravenous steroids
order an MRI of the hip
send her for an arthrogram of the hip
A 31-year-old woman presents to your office complaining of flank pain and fever for the past 2 days.
She has a history of multiple urinary tract infections. She has suffered 2 in the previous 18 months
that were treated successfully with trimethoprim-sulfamethoxazole. She is sexually active and her last
menstrual period was 6 days ago. She takes no routine medications. Her temperature is 39.5 oC
(103.1 F), blood pressure is 130/65 mm Hg, and pulse is 110/min. She has prominent right
costovertebral angle (CVA) tenderness. The most appropriate management of this patient is to
admit her to the intensive care unit
initiate therapy with ampicillin and gentamicin
obtain blood cultures and send her home pending results
obtain urine cultures and initiate therapy with ampicillin and gentamicin
obtain urine cultures and send her home pending results
A 37-year-old alcoholic patient with multiple organ failure is found to have an arterial pH of 7.15.
The anion gap is determined to be 22 mEq/L (normal is 10 to 14 mEq/L). All of the following
arelikely causes of this disorder EXCEPT
ammonium chloride ingestion
renal failure
actate accumulation
D.
E.
22.
A.
B.
C.
D. *
E.
23.
A.
B.
C.
D. *
E.
24.
A.
B.
C.
D. *
E.
25.
A.
B.
C. *
D.
E.
26.
A.
B. *
C.
D.
E.
27.
A.
B.
C.
D. *
diabetic ketosis
methanol ingestion
A 27-year-old college freshman complains of dysuria and urinary frequency. Urinalysis reveals 8to
10 WBCs per high-power field and numerousgram-negative bacteria. All of the following statements
concerning this disease are true EXCEPT that
a single dose of an antibiotic may be sufficient treatment
pregnant women with bacteriuria should be treated, even if asymptomatic
patients with flank pain or fever should be treated for 10 to 14 days
patients with indwelling catheters should receive long-term suppression
radiological investigation is appropriate formales of any age after their first infection
A 28-year-old man has the acute onset of colicky pain in the left costovertebral angle radiating into
the groin, as well as gross hematuria. Abdominal x-ray discloses a stone in the left ureter. All of the
following are true statements concerning this disease EXCEPT that
the majority of renal stones are radiopaque
radiolucent stones are usually composed of uric acid
staghorn calculi are associated with alkaline urine
radiopaque stones usually contain cystine
urate stones are associated with acidic urine
The 39-year-old patient described above spontaneously passes the stone, which is found to contain
calcium oxalate. The most likely cause of this stone is
chronic urinary tract infections
vitamin D excess
primary hyperparathyroidism
idiopathic hypercalciuria
renal tubular acidosis
A 71-year-old woman is receiving parenteral methicillin for leg cellulitis. Over 2 days she develps
macroscopic hematuria, oliguria, and marked deterioration in renal functioning. Features suggestive
of methicillin-induced acute interstitial nephritis include all of the following EXCEPT
fever and arthralgias
RBC casts in the urine
eosinophils in the urine
return of renal functioning on discontinuing the drug
return of renal functioning after prednisone therapy
A 39-year-old woman presents with indurationand atrophy of the fingertips and is diagnosed as
having systemic sclerosis. Visceral involvement may lead to all of the following EXCEPT
esophageal hypomotility
obstructive lung disease
congestive heart failure
bronchogenic carcinoma
renovascular hypertension
At a routine company physical examination, anasymptomatic 46-year-old man is found to have a BP
of 150/110 mm Hg, but no other abnormalities are present. What do you do next?
reassure the patient and repeat the physical examination in 12 months
order an outpatient hypertensive pyelogram
initiate antihypertensive therapy
obtain repeated BP recordings in your office and/or the patient's home or work site
E.
28.
A.
B.
C. *
D.
E.
29.
A. *
B.
C.
D.
E.
30.
A.
B. *
C.
D.
E.
31.
A.
B.
C.
D. *
E.
32.
A.
B. *
C.
D.
E.
hospitalize the patient for renal arteriogra phy
A 72-year-old man has the sudden onset of suprapubic pain and oliguria. His temperature is 38.0°C
(100.4°F), pulse is 100/minute, respirations are 12/minute, and BP is 110/72 mm Hg. Abdominal
examination is only remarkable for a tender, distended urinary bladder. Immediate management of
this patient should be to
obtain a flat plate of the abdomen
perform abdominal ultrasonography
insert a urethral catheter
administer furosemide IV
obtain an IVP
A 40-year-old man complained of headache in occipital area. On physical examination, the skin was
pale; there was face and hand edema, blood pressure of 170/130 mm Hg. On EchoCG, there was
concentric hypertrophy of the left ventricle. Ultrasound examination of the kidneys revealed thinned
cortical layer. Urinalysis showed proteinuria of 3.5 g/day. What is the diagnosis?
Chronic glomerulonephritis
Essential arterial hypertension
Chronic pyelonephritis
Polycystic disease of the kidneys
Chronic renal failure
A man, aged 25, presents with facial edema, moderate back pains, body temperature of 37,5 С, BP
180/100 mmHg, hematuria [ up to 100 in v/f], proteinuria [2,0 g/L], hyaline casts - 10 in v/f., specific
gravity -1020. The onset of the disease is probably connected with acute tonsillitis 2 weeks ago. The
most likely diagnosis is:
Acute pyelonephritis
Acute glomerulonephritis
Cancer of the kidney
Urolithiasis
Chronic glomerulonephritis
A 45-year-old woman who has had slowly progressive renal failure begins to complain of increasing
numbness and prickling sensations in her legs. Examination reveals loss of pinprick and vibration
sensation below the knees, absent ankle jerks, and impaired pinprick sensation in the hands. Serum
creatinine concentration, checked during her most recent clinic visit, is 0,790 mmol/L (8.9 mg/dL).
The woman's physician should now recommend
a therapeutic trial of phenytoin
a therapeutic trial of pyridoxine (vitamin B6)
a therapeutic trial of cyanocobalamin (vitamin B12)
initiation of renal replacement therapy
neurologic referral for nerve conduction studies
A 25 -year-old woman complained of edema on the face and legs, elevation in blood pressure up to
160/100 mm Hg, and weakness. She fell ill 3 weeks after sore throat. On urinalysis, protein of 0.5
g/L, erythrocytes of 17 – 20/field, leukocytes of 2 – 3/field, erythrocyte casts. What treatment should
be initiated after establishing of the exact diagnosis?
Heparin
Penicillin OS
Tetracycline
Dipyridamole
Ciprofloxacine
33.
A. *
B.
C.
D.
E.
34.
A.
B.
C.
D. *
E.
35.
A.
B.
C.
D.
E. *
36.
A.
B.
C.
D. *
E.
37.
A.
B.
C.
D.
E. *
A 29-year-old woman is critically ill. The illness was manifested by oliguria, edema, severe
headache. Pasternatsky’s sigh is negative in both sides. On lab exam, WBC of 10.000/mcL; on
urinalysis protein of 1,6g/L, erythrocyturia. Your preliminary diagnosis.
Acute pyelonephritis
Acute glomerulonephritis
Cancer of the kidney
Urolithiasis
Chronic glomerulonephritis
A 31-year-old man with a history of preceding tonsillitis complained of headache in occipital area,
edema. On physical examination, the skin was pale; there was face and extremitas edema, blood
pressure of 140/90 mm Hg. On EchoCG, there was concentric hypertrophy of the left ventricle.
Ultrasound examination of the kidneys revealed thinned cortical layer. Urinalysis showed proteinuria
of 3.5 g/day, hematurea. The level of protein in blood was 58 g/l. What treatment will be the most
effective?
ceftriaxon
enalapril
furosemid
prednisolone
dicinone
A 20- year-old patient with a history of preceding streptococcal infection complains of malaise,
headache, anorexia, subfebrile fever. On exam, mild generalized edema, BP of 150/90 mm Hg, HR of
100/min, RR of 20/min, the urinalysis showed increased protein, red cell casts and hyaline casts.
What is your diagnosis?
Chronic glomerulonephritis
Bacterial endocarditis
Rheumatic fever
Acute pyelonephritis
Acute glomerulonephritis
A 55-year-old man undergoes intravenous pyelography (IVP) as part of a workup for hypertension. A
3-cm solitary radiolucent mass is noted in the left kidney; the study otherwise is normal. The man
complains of no symptoms referable to the urinary tract, and examination of urinary sediment is
within normal limits. Which of the following studies should be performed next?
Repeat intravenous pyelography in 6 months
Early-morning urine collections for cytology (three samples)
Selective renal arteriography
Renal ultrasonography
CT scanning (with contrast enhancement) of the left kidney
A 22- year-old patient with a history of preceding streptococcal infection complains of malaise,
headache, anorexia, subfebrile fever. On exam, mild generalized edema, BP of 150/90 mm Hg, HR of
100/min, RR of 20/min, the urinalysis showed increased protein, red cell casts and hyaline casts.
What is your diagnosis?
Bacterial endocarditis
Essential hypertension
Rheumatic fever
Acute pyelonephritis
Acute glomerulonephritis
38.
A.
B. *
C.
D.
E.
39.
A.
B.
C. *
D.
E.
40.
A.
B.
C.
D.
E. *
41.
A.
B.
C.
D. *
E.
A 29-year-old woman is critically ill. The illness was manifested by oliguria, edema, severe
headache. Pasternatsky’s sigh is negative in both sides. On lab exam, WBC of 10.000/mcL; on
urinalysis protein of 1,6g/L, erythrocyturia. Your preliminary diagnosis.
Acute pyelonephritis
Acute glomerulonephritis
Cancer of the kidney
Urolithiasis
Chronic glomerulonephritis
The patient after acute respiratory viral infection [10th day of disease] has complaints on headache,
oliguria, cola-colour urine. Urinalysis – hematuria [100-200 RBC in eyeshot spot], specific gravity –
1002. The blood creatinin level is 0,18 mmol/l, potassium level - 5,4 mmol/l. Make the previous
diagnosis.
Acute interstitial nephritis
Acute renal failure
Acute glomerylonephritis
Acute cystitis
Acute renal colic
An 18-year-old white man is seen in your office because of blood on urine dipstick analysis during a
sports physical. He has no history of gross hematuria, recent fever or chills, or upper respiratory
illness. He has been in good health. He takes cetirizine for seasonal allergies. He has no family
history of renal disease. Review of systems is negative. On examination, blood pressure is 140/98
mm Hg, weight is 80 kg (176 Ib), and height is 170 cm (67’). He has a muscular build. The
remainder of the physical examination is normal. Repeated urinalysis at2 p.m. reveals a pH of 5.0,
specific gravity of 1.015,2+ blood, 2+ protein. Which test is the best option to assist in diagnosis?
Renal computed tomography
Renal ultrasonography
Cystoscopy
Magnetic resonance angiography of renal arteries
Urinary protein-to-creatinine ratio
A 19-year-old woman has painless hematuria. Five days ago, she developed a sore throat, fever
(temperature, 37.8 °C [100 °F]), and dry cough. Four days ago, she noted painless gross hematuria on
two occasions. Six months ago, at the time of a college physical examination, she was told of
microscopic hematuria and advised to have this rechecked. She does not use recreational drugs. She
has taken acetaminophen for the past several days. At presentation, the patients sore throat is
improved and she no longer has fever. On examination, she is alert and in no distress. Blood pressure
is 135/85 mm Hg, pulse rate is 78/min, and temperature is 37 °C (98.6 °F). Skin examination is
normal, and the fundi are normal. No exudate is present in the pharynx. Neck examination shows no
significant lymphadenopathy. The lungs are clear. The heart is in regular sinus rhythm, with no
murmur or rub. Abdominal examination is normal. The joints are normal. Laboratory studies:
Leukocyte count -9400/?L. Hemoglobin -13.5 g/dL. Hematocrit -39%. Blood urea nitrogen -12
mg/dL. Serum creatinine -1.0 mg/dL. Serum antinuclear antibodies Negative. Serum C3 90 mg/dL.
Serum C4 -20 mg/dL. Urine protein :creatinine ratio -0.5. Urinalysis pH 5.5; specific gravity - 1.012;
trace proteinuria, 3+ hematuria. Urine microscopy shows red blood cells, occasional red blood cell
casts, and 1 to 3 Ieukocytes/hpf. What is the most likely cause of renal disease in this patient?
Lupus nephritis
Post-streptococcal glomerulonephritis
Acute interstitial nephritis
IgA nephritis
Renal papillary necrosis
42.
A.
B.
C.
D. *
E.
43.
A.
B. *
C.
D.
E.
44.
A.
B.
C.
D.
E. *
A 45-year-old man is hospitalized after 5 days of severe diarrhea and anorexia. He has a history of
renal failure secondary to chronic glomerulonephritis and received a living related renal transplant 2
years earlier. He takes cyclosporine, mycophenolate mofetil, and prednisone, as well as amlodipine to
treat hypertension. He has had no rejection episodes or opportunistic infections. The remainder of his
history is unremarkable. On admission, blood pressure is 100/70 mm Hg and pulse rate is 100/min
while supine; blood pressure is 80/50mm Hg and pulse rate is 120/min while standing. Body
temperature is 37.5°C (99.5 °F). Physical examination reveals a thin man in no distress. Mucus
membranes are dry, and neck veins are flat at 30-degree elevation. Cardiopulmonary examination is
unremarkable except for resting tachycardia. Abdominal palpation reveals diffuse tenderness without
peritoneal signs. No peripheral edema is present. Laboratory studies: Blood urea nitrogen
40
mg/dL Serum creatinine - 1.0 mg/dL. Serum sodium - 134 mmol/L. Serum potassium - 3.8 meq/L.
Serum chloride - 108 meq/L. Serum bicarbonate - 16 meq/L. Which urinalysis results best
No
protein, to
pHthis
7, specific
1.005
correspond
patients gravity
condition?
No protein, pH 7, specific gravity 1.030
1 + protein, pH 5, specific gravity 1.005
1 + protein, pH 5, specific gravity 1.030
No protein, pH 5, specific gravity, 1.005
A 17-year-old student presents with a 2-week history of periorbital and lower extremity swelling that
developed over 3 to 4 days. The patient had been well until this point. He takes no medications and
has no history of recreational drug use. On examination, blood pressure is 110/70 mm Hg, pulse rate
is 92/min, and temperature is 37 °C (98.6 °F). Skin examination is normal. He has marked periorbital
edema. No jugulovenous distention is present. The lungs are clear, and the heart is in regular sinus
rhythm, with no murmur or gallop. The abdomen is nontender, and the liver and spleen are not
palpable. Scrotal edema is present, as is 4+ edema of the legs and thighs. Laboratory studies:
Complete blood count Normal Blood urea nitrogen 25 mg/dL Serum creatinine 1.2 mg/dL Serum
total protein 6.5 g/dL Serum albumin
2.0 g/dL Urine protein:creatinine ratio 18 Urinalysis pH
6.0; specific gravity 1.020; 4+ proteinuria, trace hematuria; many hyaline, granular, and fatty casts
Renal biopsy shows 15 glomeruli. Light microscopy is normal. Immunofluorescence shows
nonspecific staining for C3. Electron microscopy shows fusion of podocyte foot processes. What is
the renal diagnosis in this patient?
Membranous nephropathy
Minimal change disease
Membranoproliferative glomerulonephritis
Focal and segmental glomerulonephritis
Alports disease
The condition of a 50-year-old obese woman with a 5-year history of mild hypertension controlled by
a thiazide diuretic is being evaluated because proteinuria was noted during her routine yearly medical
visit. Physical examination disclosed a height of 167.6 cm (66 in.), weight 91 kg (202 lb), blood
pressure 130/80 mmHg, and trace pedal edema. Laboratory values are as follows:Serum creatinine:
106 jxmol/L (1.2 mg/dL) Creatinine clearance: 87 mL/min Urinalysis: pH 5.0; specific gravity 1.018;
protein 3 + ; no glucose; occasional coarse granular cast Urine protein excretion: 5.9 g/d The results
of a renal biopsy are :sixty percent of the glomeruli appeared sclerotic as “halfmoons”; the remainder
were unremarkable. The most likely diagnosis is
hypertensive nephrosclerosis
focal and segmental sclerosis
minimal-change (nil) disease
membranous glomerulopathy
crescentic glomerulonephritis
45.
A.
B. *
C.
D.
E.
46.
A.
B.
C. *
D.
E.
47.
A.
B. *
C.
D.
E.
A 33-year-old obese woman has four healthy children from three previous uncomplicated term
pregnancies. She has a strong family history of hypertension but has never had hypertension in the
presence or absence of pregnancy. She was first seen for prenatal care by a physician 2 months after
her last menstrual period. Her pregnancy had been uneventful, and she had gained 3.6 kg (8 Ib). She
was seen 3 months after her last menstrual period. Blood pressure was 158/94 mm Hg, and she had
trace edema. Laboratory values at the time were as follows: Hematocrit 33.4%. Blood urea nitrogen 9
mg/dL. Serum creatinine 0.4 mg/dL. Serum uric acid 1 mg/dL. 24-hour urinary protein - 100 mg.
Creatinine clearance - 150 mL/min. Urinalysis Trace proteinuria by dipstick. The patient is seen 1
week later. Her blood pressure is 162/92 mm Hg. She has trace proteinuria, and laboratory values
have not changed significantly. What is the best course of action?
Start ramipril therapy
Start methyldopa therapy
Start atenolol therapy
Check her blood pressure in 1 week and advise the patient that better control of blood pressure will
reduce her risk for preeclampsia
Advise termination of pregnancy
A 31-year-old white woman with diabetes presents for management of her hypertension. She
developed type 1 diabetes mellitus at 9 years of age and claims that the condition is under reasonable
control. However, she knows that she has diabetic retinopathy and proteinuria. Her blood pressure
had been elevated in the range of 140 to 150/90 mm Hg at the last three visits to her family physician.
The patient does not smoke, drink alcohol, or use recreational drugs. She adheres to a no-added-salt
diet. The only medication that she takes is insulin. On examination, blood pressure is 152/90 mm Hg
seated and 146/88 mm Hg standing. Body weight is 82 kg (181 Ib). The examination is normal other
than nonproliferative diabetic retinopathy. What is the most appropriate antihypertensive therapy for
this patient?
Dihydropyridine calcium channel blocker
Angiotensin receptor blocker
Angiotensin-converting enzyme inhibitor
Intensified lifestyle modification
none of them
A 59-year-old white male computer analyst is referred for evaluation of hypertension (185/95 mm
Hg) discovered during a blood pressure screening at his workplace. The patient states that he is well
and has not seen a physician in many years. He describes himself as “a fitness freak, as he is an
active jogger, abstains from alcohol, and limits his salt and fat intake. He denies any knowledge of
hypertension, cardiovascular disease, renal disease, or diabetes mellitus. He takes no medications
regularly. Family history is significant in that his father was known to be hypertensive and died of a
stroke at 64 years of age. His older brother is being treated for hypertension. On examination, the
patient appears well, with a blood pressure of 174/98 mm Hg while seated and standing. Body weight
is 71 kg (157 Ib), and height is 178 cm (70). Optic fundus examination is significant for grade II
hypertensive retinopathy. The remainder of the examination is normal. Complete blood count,
electrolyte panel, blood urea nitrogen level, creatinine concentration, thyroid-stimulating hormone
level, and results of urinalysis are normal. Electrocardiography demonstrates normal sinus rhythm
with left ventricular hypertrophy. To reduce the patient’s cardiovascular morbidity and mortality,
Doxazosin
which therapy would you prescribe?
Losartan
Atenolol
Hydralazine
Niphedipdin
48.
A.
B.
C.
D. *
E.
49.
A.
B. *
C.
D.
E.
50.
A.
B. *
C.
D.
Two weeks after therapy is initiated in the 31-year-old white woman with diabetes presents for
management of her hypertension Who developed type 1 diabetes mellitus at 9 years of age and
claims that the condition is under reasonable control, her blood pressure decreases from 150/90 mm
Hg to 128/80mm Hg, and she feels well. Repeated laboratory testing reveals an increase in serum
creatinine concentration from 1.9 mg/dL to 2.1 mg/dL. The potassium concentration is 4.2 mgld L.
Which of the following is the most appropriate course of action?
Discontinue antihypertensive therapy
Perform noninvasive screening for possible renal artery stenosis
Perform renal angiography
Continue antihypertensive therapy and monitor kidney function
none of them
A 50-year-old previously healthy man presents with acute shortness of breath, right pleuritic chest
pain, and pink sputum. He has no fever or chills. For the past month, he has noticed swelling of the
ankles. He has had no prior symptoms of dyspnea on exertion, paroxysmal nocturnal dyspnea, or
orthopnea. He has had no joint pain or skin rash. He takes no medications and does not smoke. On
examination, the patient is alert but in mild respiratory distress. Blood pressure is 152/100 mm Hg,
pulse rate is 100/min, and respiratory rate is 22/min. There is no jugulovenous distention and no rash.
The lungs are clear. Cardiac examination shows regular sinus rhythm, with no gallop, murmur, or
rub. Abdominal examination shows no hepatosplenomegaly. The patient has 2+ to 3+ edema of the
ankles and legs. Laboratory studies: Leukocyte count 8,5 109L Platelet count – 200 109 /L.
Hemoglobin - 14 g/dL. Hematocrit - 42%. Blood urea nitrogen 18 mg/dL. Serum creatinine - 1.4
mg/dL. Serum sodium - 138 meq/L. Serum chloride - 105 meq/L. Serum potassium - 4.1 meq/L.
Serum bicarbonate - 23 meq/L. Serum total protein - 5.9 g/dL. Serum albumin - 1 .9 g/dL. Serum
cholesterol - 300 mg/dL. Urinalysis Specific gravity 1.015; 4+ proteinuria; oval fat bodies, granular
casts, 3-5 erythrocytes/hpf 24-hour urine protein 10g Arterial blood gasespH 7.46; Po280 mm Hg;
Pco233 mm Hg Serum C380 mg/dL Serum C425 mg/dL Antinuclear antibodies Negative Chest
radiography shows clear lung fields and normal heart size. Ventilation-perfusion scanning reveals a
mismatch in the right lower lung field. Sonography of lower extremities shows right femoral vein
thrombosis. Which renal disease is most likely in this patient?
Goodpastures syndrome
Membranous nephropathy
Wegener’s granulomatosis
Lupus nephritis
Microscopic polyangiitis
A 59-year-old white male computer analyst is referred for evaluation of hypertension (185/95 mm
Hg) discovered during a blood pressure screening at his workplace. The patient states that he is well
and has not seen a physician in many years. He describes himself as “a fitness freak, as he is an
active jogger, abstains from alcohol, and limits his salt and fat intake. He denies any knowledge of
hypertension, cardiovascular disease, renal disease, or diabetes mellitus. He takes no medications
regularly. Family history is significant in that his father was known to be hypertensive and died of a
stroke at 64 years of age. His older brother is being treated for hypertension. On examination, the
patient appears well, with a blood pressure of 174/98 mm Hg while seated and standing. Body weight
is 71 kg (157 Ib), and height is 178 cm (70). Optic fundus examination is significant for grade II
hypertensive retinopathy. The remainder of the examination is normal. Complete blood count,
electrolyte panel, blood urea nitrogen level, creatinine concentration, thyroid-stimulating hormone
level, and results of urinalysis are normal. Electrocardiography demonstrates normal sinus rhythm
with left ventricular hypertrophy. To reduce the patient’s cardiovascular morbidity and mortality,
Doxazosin
which therapy would you prescribe?
Losartan
Atenolol
Hydralazine
E.
51.
A.
B. *
C.
D.
E.
52.
A.
B.
C.
D.
E. *
53.
A.
B. *
C.
D.
Niphedipdine
A 58-year-old black woman presents for routine follow-up of diabetes mellitus and hypertension. She
feels well but states that she stopped taking verapamil because of constipation. Current medications
include glipizide, pravastatin, and aspirin; evidence of drug intolerance includes
angiotensin-converting enzyme inhibitor cough. On examination, blood pressure is 156/92 mm Hg
seated and standing. Except for the patient’s findings for background diabetic retinopathy, the
remainder of the examination is normal. Recent laboratory values are a serum creatinine
concentration of 1.6 mg/dL, 24-hour urinary protein excretion of 1.5 g/d, and creatinine clearance of
45 mL/min. On the basis of recent evidence, what is the most efficacious therapy to slow the
Angiotensin-converting
enzyme
progression of the patient’s
type inhibitor
2 diabetic nephropathy?
Angiotensin receptor blocker
Dihydropyridine calcium antagonist
?-Blocker
none of them
A 66-year-old man has had a several-week history of fatigue and ankle swelling. His appetite is
normal, but his body weight has increased 4.5 kg (10 Ib). There is no history of exertional dyspnea,
paroxysmal nocturnal dyspnea, or orthopnea. He has had hypertension for 10 years, and diabetes was
diagnosed 4 months ago. He takes hydrochlorothiazide, 25 mg/d, and metoprolol, 50 mg/d. On
examination, the patient is alert and in no distress. Blood pressure is 120/75 mm Hg, pulse rate is
82/min, and temperature is 36.9 °C (98.4 °F). Skin examination is normal. The lungs are clear.
Cardiac examination shows regular sinus rhythm, with no murmur or gallop. The abdomen is slightly
protuberant with shifting dullness, but no hepatosplenomegaly is noted. There is 4+ edema of the legs
and thighs and 1+ edema of the sacrum. Laboratory studies: Complete blood count Normal
Hemoglobin A1c 7.4%. Blood urea nitrogen - 20 mg/dL. Serum creatinine - 0.9 mg/dL. Serum
sodium - 141 meq/L. Serum chloride - 104 meq/L. Serum potassium - 4.3 meq/L. Serum total protein
- 4.4 g/dL. Serum albumin - 1.7 g/dL. Serum cholesterol - 376 mg/dL. 24-hour urine protein 8.5 g.
Urinalysis pH 5.5; specific gravity 1 .020; protein 4+, trace hemoglobin Urine microscopy shows
many granular and hyaline casts and oval fat bodies. Monoclonal protein is identified on urine
immunoelectrophoresis. Plasma immunoelectrophoresis shows an IgG level of 452 mg/dL, IgA of
284 mg/dL, 1gM of 122 mg/dL, K of 550 mg/dL, and 2 of 193 mg/dL with a homogenous M band.
What renal disease is most likely in this patient?
Idiopathic membranous glomerulopathy
Diabetic nephropathy
Focal and segmental glomerulosclerosis
AA amyloidosis
AL amyloidosis
A 33-year-old obese woman has four healthy children from three previous uncomplicated term
pregnancies. She has a strong family history of hypertension but has never had hypertension in the
presence or absence of pregnancy. She was first seen for prenatal care by a physician 2 months after
her last menstrual period. Her pregnancy had been uneventful, and she had gained 3.6 kg (8 Ib). She
was seen 3 months after her last menstrual period. Blood pressure was 158/94 mm Hg, and she had
trace edema. Laboratory values at the time were as follows: Hematocrit - 33.4%. Blood urea nitrogen
- 9 mg/dL. Serum creatinine - 0.4 mg/dL. Serum uric acid - 3.1 mg/dL. 24-hour urinary protein - 100
mg. Creatinine clearance - 150 mL/min. Urinalysis Trace proteinuria by dipstick. The patient is seen
1 week later. Her blood pressure is 162/92 mm Hg. She has trace proteinuria, and laboratory values
have not changed significantly. What is the best course of action?
Start ramipril therapy
Start methyldopa therapy
Start atenolol therapy
Check her blood pressure in 1 week and advise the patient that better control of blood pressure will
reduce her risk for preeclampsia
E.
54.
A.
B. *
C.
D.
E.
55.
A.
B.
C.
D. *
E.
56.
A.
B.
C. *
D.
E.
57.
A. *
B.
C.
D.
E.
Advise termination of pregnancy
A 60-year-old woman with adult polycystic kidney disease is seen urgently in the office for high
fever. The illness started abruptly and involves chills and dysuria. She has had hypertension for the
past 5 years, treated with quinapril and hydrochlorothiazide. She has lost approximately 8 kg (17 Ib)
of weight over the last 3 months. On physical examination, the patient appears thin and frail. Body
weight is 48 kg (106 Ib). Blood pressure is 90/70 mm Hg, pulse rate is 110/min, respiratory rate is
24/min, and body temperature is 39 °C (102 °F). The kidneys are palpable bilaterally, and she has
right costovertebral angle tenderness. Serum creatinine concentration is 1.1 mg/dL, and urinalysis
shows pyuria and bacteriuria. The patient is admitted and prescribed intravenous ampicillin and
gentamicin to treat pyelonephritis. Why does the dosage of antibiotic need to be adjusted in this
The
infection is in a cyst
patient?
The glomerular filtration rate is reduced
The patient is septic
The patient has hypertension
none of them
A 46-year-old man with chronic kidney disease secondary to biopsy-proven focal and segmental
glomerulosclerosis returns for routine follow-up. The hematocrit is 28%, and potentially correctable
causes of anemia have been excluded. Therapy with recombinant human erythropoietin is
recommended. In patients with chronic kidney disease and pre-end-stage renal disease, what is a
benefit of therapy with erythropoietin to effectively treat anemia?
Reduced mortality
Decreased cardiovascular event rates
Normalization of hypertension
Regression of left ventricular hypertrophy
none of them
A 42-year-old man complained of headache in occipital are. On physical examination, the skin was
pale; there was face and hand edema, blood pressure of 170/135 mm Hg. On EchoCG, there was
concentric hypertrophy of the left ventricle Ultrasound examination of the kidneys revealed thinned
cortical layer. Urinalysis showed proteinuria of 3.6 g/day. What is the diagnosis?
Essential arterial hypertension
Chronic pyelonephritis
Chronic glomerulonephritis
Allergic interstitial nephritis
Goodpastures syndrome
A 21- year-old patient with a history of preceding streptococcal infection complains of malaise,
headache, anorexia, subfebrile fever. On exam, mild generalized edema, BP of 150/90 mm Hg, HR of
100/min, RR of 20/min, the urinalysis showed increased protein, red cell casts and hyaline casts.
What is your diagnosis in this patient?
Acute glomerulonephritis
Acute pyelonephritis
Chronic renal failure
Acute tubular necrosis
Rheumatic fever
58.
A.
B. *
C.
D.
E.
59.
A.
B. *
C.
D.
E.
60.
A.
B. *
C.
D.
E.
61.
A.
B. *
C.
A 58-year-old black woman presents for routine follow-up of diabetes mellitus and hypertension. She
feels well but states that she stopped taking verapamil because of constipation. Current medications
include glipizide, pravastatin, and aspirin; evidence of drug intolerance includes
angiotensin-converting enzyme inhibitor cough.
On examination, blood pressure is 156/92 mm Hg seated and standing. Except for the patient’s
findings for background diabetic retinopathy, the remainder of the examination is normal. Recent
laboratory values are a serum creatinine concentration of 1.6 mg/dL, 24-hour urinary protein
excretion of 1.5 g/d, and creatinine clearance of 45 mL/min. On the basis of recent evidence, what is
the most efficacious therapy to slow the progression of the patient’s type 2 diabetic nephropathy?
Angiotensin-converting enzyme inhibitor
Angiotensin receptor blocker
Dihydropyridine calcium antagonist
?-Blocker
None of them
A 39-year-old black man presents for hypertension discovered during a pre-employment
examination. He is healthy but has a family history of hypertension in both parents and two siblings.
He has no history of cardiovascular disease and does not use tobacco, alcohol, or recreational drugs.
He is taking no medications. The patient appears well. Height is 173 cm (68), body weight is 78 kg
(172 Ib), and blood pressure is 160/1 02 mm Hg seated and standing. The physical examination is
otherwise normal. A complete blood count and electrolyte panel are normal. The serum creatinine
concentration is 1 .8 mg/dL, and urinalysis reveals 2+ proteinuria. Which is the most appropriate
antihypertensive therapy for this patient?
Intensive lifestyle modification
Angiotensin-converting enzyme inhibitor
Diuretic
Nondihydropyridine calcium channel blocker
None of them
A 57-year old woman comes to the emergency department because of a "very high fever." She has
diabetes mellitus and hemodialysis-dependent renal failure. She also has hypertension and is
status-post total abdominal hysterectomy. She is frail appearing and diaphoretic. Her blood pressure
is 170/90 mm Hg and temperature is 38.3 C (101.0 F). Her neck is supple without any specific
meningismus. She has a Tesio catheter in her left subclavian vein. Her lungs are clear and she has no
costovertebral angle tenderness. Her laboratory studies show a white blood cell count of 23,000/mm3
and a hematocrit of 31%. Her urinalysis is dipstick negative for white blood cells. The most
appropriate next step in management is to
begin antibiotic therapy with gentamycin
begin antibiotic therapy with vancomycin and gentamycin
order urinalysis analysis and culture
perform a lumbar puncture and send CSF for analysis and culture
schedule emergent surgical removal of her Tesio catheter
An 18-year-old male high school student presents to the emergency department with confusion,
nausea, headache, and decreased vision after a camping trip. The patient’s friends state that he
became ill 12 to 14 hours ago. Laboratory studies: Blood urea nitrogen
14 mg/dL Serum
creatinine
1.0 mg/dL Serum sodium
140 meq/L Serum chloride 100 meq/L Serum
potassium
4 meq/L Serum bicarbonate 12 meq/L Serum glucose
108 mg/dL Measured
serum osmolality
326 Serum ketones Negative Serum lactate
0.7 meq/L Arterial blood
gases pH 7.29, PCO2 26 mm Hg What ingestion best explains the acid-base abnormalities?
Ethanol
Methanol
Isopropyl alcohol
D.
E.
62.
A.
B. *
C.
D.
E.
63.
A. *
B.
C.
D.
E.
64.
A.
B.
C.
D. *
E.
Salicylate
None of them
Two weeks after therapy is initiated in the 31-year-old white woman with diabetes presents for
management of her hypertension Who developed type 1 diabetes mellitus at 9 years of age and
claims that the condition is under reasonable control, her blood pressure decreases from 150/90 mm
Hg to 128/80mm Hg, and she feels well. Repeated laboratory testing reveals an increase in serum
creatinine concentration from 1.9 mg/dL to 2.1 mg/dL. The potassium concentration is 4.2 mgld L.
Which of the following is the most appropriate course of action?
Start bicarbonate supplementation and add insulin to control blood glucose
Continue antihypertensive therapy and monitor kidney function
Institute plasma exchange to treat acidemia
Initiate ethanol drip and continue insulin supplementation
Perform emergency contrast computed tomography of the abdomen and pelvis
A 72-year-old white man returns for a follow-up visit subsequent to admission to another hospital for
hypertension. He was seen in the local emergency department for severe musculoskeletal back pain,
where the treating physician noted elevated blood pressure (200/92 mm Hg) and a serum creatinine
concentration of 1.6 mg/dL. Results of other laboratory tests were normal. The patient was admitted
for evaluation and management of back pain, and the attending physician obtained additional studies
relating to the patients hypertension. Renal ultrasonography was negative for calculus, mass, or
obstruction; kidney size was 11 cm on the right and 12 cm on the left. Renal artery duplex
ultrasonography was suggestive of right renal artery stenosis. Renal angiography revealed a normal
left renal artery and 50% stenosis in the right renal artery. Analysis of renal vein renin activity
showed a low inferior vena cava value of 1.5 g/L/h, right renal vein value of 2.0 g/L/h, left renal vein
value of 2.0 g/L/h, and a high inferior vena cava value of 2.0 g/L/h. The plasma renin activity is 1 .0
mg/LIh, and the plasma aldosterone level is 8.0 ng/dL.The thyroid-stimulating hormone level is 1.0
U/mL. Review of your office records confirms that the patient has a 22-year history of hypertension
controlled with a -blocker and diuretic therapy. The serum creatinine concentration has been stable at
1.6 mgldL for more than 3 years, and urinalysis shows 1 + proteinuria. What is the cause of this
Primary
hypertension
man’s hypertension?
Hypothyroidism
Primary hyperaldosteronism
Renovascular hypertension
Pheochromocytoma
An 18-year-old white man is seen in your office because of blood on urine dipstick analysis during a
sports physical. He has no history of gross hematuria, recent fever or chills, or upper respiratory
illness. He has been in good health. He takes cetirizine for seasonal allergies. He has no family
history of renal diseasReview of systems is negativOn examination, blood pressure is 140/98 mm Hg,
weight is 80 kg (176 Ib), and height is 170 cm (67’). He has a muscular builThe remainder of the
physical examination is normal. Repeated urinalysis at2 p.m. reveals a pH of 5.0, specific gravity of
1.015,2+ blood, 2+ protein. Which test is the best option to assist in diagnosis?
Renal ultrasonography
Cystoscopy
Magnetic resonance angiography of renal arteries
Urinary protein-to-creatinine ratio
Renal computed tomography
65.
A.
B. *
C.
D.
E.
66.
A.
B. *
C.
D.
E.
67.
A.
B.
C.
D. *
E.
A 34-year-old pregnant woman with a 5-year history of biopsy-diagnosed hypertensive nephropathy
has been followed in obstetric clinic for 3 months after her last menstrual period. One year ago, her
serum creatinine concentration was 1 .6 mg/dL. Her pregnancy has been uneventful. Her blood
pressure has been well controlled on a combination of methyldopa and hydralazine and is currently
130/85 mm Hg. She has trace edema. Laboratory studies: Hematocrit
37% Leukocyte count
Normal Platelet count
Normal Peripheral smear No schistocytes Blood urea nitrogen 14
mg/dL Serum creatinine 1.8 mg/dL Serum uric acid 4.9 mg/dL Urinalysis Specific gravity, 1.010;
urinary protein 4+ by dipstick; no glucosuria, hematuria, or ketonuria Microscopic urine examination
shows rare broad casts. Liver function tests are normal. Which one of the following statements about
She
has developed
preeclampsia.
the patients
course is
true?
The course is most consistent with progression of her chronic renal disease.
She has developed microangiopathic hemolytic anemia.
She has developed prerenal azotemia.
Her blood pressure is likely to improve during the course of her pregnancy.
A 58-year-old nun comes to your office because of lethargy, mild nausea, and weakness for the past 2
weeks. Three years ago, pulmonary sarcoidosis was diagnosed by biopsy. Three months ago, the
patient began taking oral calcium (1500 mg/d) and 25-hydroxyvitamin D as treatment for
osteoporosis that was diagnosed by screening bone density testing. She has chronic hypertension that
is well controlled with metoprolol, 50 mg/d. On examination, the patient appears thin but well
nourished and is in no distress. She is oriented to time, date, and place. Blood pressure is 140/80 mm
Hg, pulse rate 80/min, temperature 37 °C (98.6°F). The thyroid is normal, and the neck veins are not
distended. The lungs are clear. Cardiac examination shows regular sinus rhythm, no murmur, and
normal first and second heart sounds. The abdomen is not tender, the liver and spleen are not
palpable, and no mass is present. There is no edema in the lower extremities, and reflexes are 1 + and
symmetrical. Laboratory studies: Hemoglobin
13.8 g/dL Hematocrit
38% Leukocyte
count 5600/?L Blood urea nitrogen 24 mg/dL Serum creatinine 2.2 mg/dL (was 1.0 mg/dL 3
months ago) Serum sodium 141 meq/L Serum potassium 4.4 meq/L Serum chloride
105 meq/L
Serum bicarbonate 24 meq/L Serum calcium
12.8 mg/dL Serum phosphorus
3.5 mg/dL
Serum parathyroid hormone 18 pg/mL Urinalysis pH 5.5; specific gravity 1.010; no proteinuria,
hematuria, or glucosuria; no cells on microscopy Serum and urine immunoglobulins showed no
monoclonal protein. A polyclonal increase in IgG is present. Renal ultrasonography demonstrates no
hydronephrosis and no calculi. What is the most likely cause of this patients acute renal failure?
Myeloma kidney
Acute interstitial nephritis
Hypercalcemia
Acute glomerulonephritis
Bilateral renal artery stenosis
A 47-year-old man calls Monday morning seeking help with “the worst headache ever” Friday night
and Saturday. The headache was associated with severe lethargy and intermittent confusion. He
recovered and has felt well for the past 24 hours. He states that he does not have fever or neurologic
or cardiovascular symptoms. His medical history is significant for hypertension and recurrent urinary
tract infections related to his known autosomal dominant polycystic kidney disease. He is concerned
because his father died of a stroke during dialysis. The serum creatinine concentration is 2.6 mg/dL.
What do you recommend for this patient?
Make an office appointment for him to see you this week
Arrange a consultation with the neurology/headache clinic
Order computed tomography of the head without contrast
Arrange urgent magnetic resonance angiography of the head
None of them
68.
A. *
B.
C.
D.
E.
69.
A.
B.
C. *
D.
E.
70.
A.
B.
C. *
D.
A 35-year-old man had HIV infection diagnosed 2 months ago. His serum creatinine concentration
was 0.6 mg/dL. Treatment with highly active antiretroviral therapy with zidovudine, lamivudine, and
abacavir was recommended, but he wished to wait before starting treatment. He is brought to clinic
by a friend who states that the patient has had fever, confusion, and disorientation for 1 day. Physical
examination reveals blood pressure 110/70 mm Hg and pulse rate 1 00/min that is regular supine and
standing. The chest is clear, without cardiac murmur or gallop, and the abdomen is normal. Moderate
bilateral lower extremity edema is present. Laboratory studies: Hemoglobin - 7.8 g/dL Leukocyte
count - 10,2000/?L Platelet count - 19,000/?L Blood urea nitrogen 37 mg/dL Serum creatinine - 2.7
mg/dL Serum sodium - 136 meq/L Serum potassium
5.2 meq/L Serum chloride - 99 meq/L
Serum bicarbonate - 22 meq/L Urinalysis Specific gravity 1.030; 3+ hematuria, trace proteinuria,
trace ketonuria, no glucosuria Urinary microscopic examination shows a few erythrocytes, but no
erythrocyte casts. The lactate dehydrogenase level is elevated. Peripheral blood smear shows many
schistocytes. What is the most likely cause of this patient’s renal failure?
Thrombotic thrombocytopenic purpura
HIV-associated nephropathy
Surreptitious ingestion of antiretroviral drugs
Outpatient acute tubular necrosis
HIV-associated immune-mediated glomerulonephritis
48-year-old white male plumber transfers to your practice after a change of insurance status. His
medical history is positive for primary hypertension without target organ damage. He has no history
of renal or prostatic disease. Laboratory values obtained from his former primary care physician
show normal results for blood urea nitrogen, serum creatinine, electrolytes, urinalysis,
prostate-specific antigen, and electrocardiography. He takes the ?-blocker doxazosin, 2 mg at
bedtime. On examination, blood pressure is 146/92 mm Hg seated and standing. Body weight is 84
kg (185 Ib). The remainder of the examination is normal. What is the appropriate course of action
regarding the patient’s antihypertensive therapy?
Increase doxazosin to 4 mg
Advise high dietary intake of calcium and potassium
Discontinue doxazosin therapy and consider an alternative agent
Advise a low-sodium diet
none of them
24-year-old graduate student has a several-week history of aching pains in knees, ankles, and elbows
and intermittent abdominal pain. Three days ago, he had two episodes of painless hematuria. He has
no fever, chills, or weight loss. There is no history of hair loss, but he has had an intermittent,
nonpruritic rash located below the knees bilaterally. There has been no recent upper respiratory
infection. He takes acetaminophen occasionally for the aching joints. He has no history of kidney
disease or intravenous drug use. On examination, the patient appears well. Weight is 84.4 kg (186
Ib). Blood pressure is 120/84 mm Hg, pulse rate is 70/min, and temperature is 37.2 °C (99 °F). No
jugulovenous distention is present. The lungs are clear, and cardiac examination shows regular sinus
rhythm with no murmur. On abdominal examination, the liver and spleen are not palpable, and no
mass or tenderness is present. Pulses in the extremities are normal and there is no edema, but the
patient has a petechial macular papular rash over the lower extremities. Laboratory studies:
Leukocyte count - 8400/?L Hemoglobin - 14.1 mgldL Hematocrit - 41 % Blood urea nitrogen - 12
mg/dL Serum creatinine
1.1 mg/dL Serum sodium - 138 meq/L Serum chloride - 104 meq/L
Serum potassium - 4.0 meq/L Serum bicarbonate - 26 meq/L Serum antinuclear antibody Negative
Serum C3 - 85 mg/dL Serum C4 - 21 mg/dL Urine protein :creatinine ratio0.2 Urinalysis Specific
gravity 1.030, pH 5.0, 1 + proteinuria, 4+ hematuria Urine microscopy reveals many dysmorphic
erythrocytes and erythrocyte casts. The lung fields are clear on chest radiography. Biopsy of skin
lesion shows IgA deposition and leukocytoclastic change. What is the most likely cause of renal
Lupus
diseasenephritis
in this patient?
Acute post—streptrococcal glomerulonephritis
Henoch-Sch nlein purpura
Allergic interstitial nephritis
E.
71.
A.
B.
C.
D. *
E.
72.
A.
B.
C. *
D.
73.
A.
B.
C.
D. *
E.
Goodpastures syndrome
A 38-year-old black man presents for hypertension discovered during a pre-employment
examination. He is healthy but has a family history of hypertension in both parents and two siblings.
He has no history of cardiovascular disease and does not use tobacco, alcohol, or recreational drugs.
He is taking no medications. The patient appears well. Height is 173 cm (68), body weight is 78 kg
(172 Ib), and blood pressure is 158/1 02 mm Hg seated and standing. The physical examination is
otherwise normal. A complete blood count and electrolyte panel are normal. The serum creatinine
concentration is 1 .8 mg/dL, and urinalysis reveals 2+ proteinuria. Which is the most appropriate
antihypertensive therapy for this patient?
Intensive lifestyle modification
Diuretic
Nondihydropyridine calcium channel blocker
Angiotensin-converting enzyme inhibitor
none of them
A 28-year-old woman presents for evaluation of recurrent kidney stones that she says ‘contain
calcium.” She estimates that she has passed four stones during the past 4 years. She currently has no
symptoms of renal colic. For several years, she has had dry eyes and dry mouth. She also describes
symptoms of Raynauds phenomenon. Crohns disease was diagnosed 10 years ago; the patient is
currently asymptomatic and passes one formed stool daily. She takes no medications. There is no
family history of renal stone disease. On examination, the patient is alert and healthy. Blood pressure
is 115/74 mm Hg, pulse rate is 72/min, and temperature is 37 °C (98.6 °F). The skin is clear, and the
joints are normal. The lungs are clear. Cardiac examination shows regular sinus rhythm and no
murmur. The liver and spleen are not palpable, and the abdomen is not tender. Plain abdominal
radiography shows multiple calcifications overlying both renal shadows. Laboratory studies:
Hemoglobin - 13.2 g/dL Hematocrit - 39% Leukocyte count - 7400/?L Blood urea nitrogen - 18
mg/dL Serum creatinine - 0.9 mg/dL Serum sodium - 138 meq/L Serum potassium - 2.8 meq/L
Serum chloride - 109 meq/L Serum bicarbonate - 19 meq/L Serum calcium - 9.1 mg/dL Serum
phosphorus - 3.2 mg/dL Urinalysis pH 6.0; specific gravity 1.020; trace hematuria, no proteinuria
Arterial blood pH 7.29 What is the most likely etiology of this patients renal stone disease?
Idiopathic hypercalciuria
Primary hyperthyroidism
Distal renal tubular acidosis
Enteric hyperoxaluria
A 40-year-old man has recurrent nephrolithiasis due to idiopathic hypercalciuria. He has had more
than 40 calcium oxalate stones in the past 5 years. He is started on hydrochlorothiazide therapy and a
low-sodium diet. During treatment, his 24-hour urinary calcium concentration decreases from 385
mg/d to 180 mg/d. No new stones have formed in the past 6 months; however, hypokalemia has
developed (serum potassium level, 2.9 meq/L). Taking the hypokalemia into account, what therapy
should the patient receive for hypercalciuric stone disease?
High-potassium diet plus hydrochlorothiazide
Acetazolamide plus hydrochlorothiazide
Magnesium oxide plus hydrochlorothiazide
Amiloride plus hydrochlorothiazide
none of them
74.
A.
B. *
C.
D.
E.
75.
A. *
B.
C.
D.
E.
76.
A. *
B.
C.
D.
E.
77.
A.
B.
C. *
75.
A 58-year-old nun comes to your office because of lethargy, mild nausea, and weakness for
the past 2 weeks. Three years ago, pulmonary sarcoidosis was diagnosed by biopsy. Three months
ago, the patient began taking oral calcium (1500 mg/d) and 25-hydroxyvitamin D as treatment for
osteoporosis that was diagnosed by screening bone density testing. She has chronic hypertension that
is well controlled with metoprolol, 50 mg/d. On examination, the patient appears thin but well
nourished and is in no distress. She is oriented to time, date, and place. Blood pressure is 140/80 mm
Hg, pulse rate 80/min, temperature 37 °C (98.6°F). The thyroid is normal, and the neck veins are not
distended. The lungs are clear. Cardiac examination shows regular sinus rhythm, no murmur, and
normal first and second heart sounds. The abdomen is not tender, the liver and spleen are not
palpable, and no mass is present. There is no edema in the lower extremities, and reflexes are 1 + and
symmetrical. Laboratory studies: Hemoglobin
13.8 g/dL Hematocrit
38% Leukocyte
count 5600/?L Blood urea nitrogen 24 mg/dL Serum creatinine 2.2 mg/dL (was 1.0 mg/dL 3
months ago) Serum sodium 141 meq/L Serum potassium 4.4 meq/L Serum chloride
105 meq/L
Serum bicarbonate 24 meq/L Serum calcium
12.8 mg/dL Serum phosphorus
3.5 mg/dL
Serumparathyroid hormone 18 pg/mL Urinalysis pH 5.5; specific gravity 1.010; no proteinuria,
hematuria, or glucosuria; no cells on microscopy Serum and urine immunoglobulins showed no
monoclonal protein. A polyclonal increase in IgG is present. Renal ultrasonography demonstrates no
hydronephrosis and no calculi. What is the most likely cause of this patients acute renal failure?
Myeloma kidney
Acute interstitial nephritis
Hypercalcemia
Acute glomerulonephritis
Bilateral renal artery stenosis
A 40-year-old man complained of headache in occipital area. On physical examination, the skin was
pale; there was face and hand edema, blood pressure of 170/130 mm Hg. On EchoCG, there was
concentric hypertrophy of the left ventricle. Ultrasound examination of the kidneys revealed thinned
cortical layer. Urinalysis showed proteinuria of 3.5 g/day. What is the diagnosis
Chronic glomerulonephritis
Essential arterial hypertension
Chronic pyelonephritis
Polycystic disease of the kidneys.
Cushing’s disease
A 29-year-old woman is critically ill. The illness was manifested by high fever, chills, sweating,
aching pain in lumbar area, a discomfort in urination, and frequent voiding. Pasternatsky’s sigh is
positive in both sides. On lab exam, WBC of 20.000/mcL; on urinalysis protein of 0.6g/L,
leukocyturia, bacteriuria. Your preliminary diagnosis.
Acute pyelonephritis
Exacerbation pf chronic pyelonephritis
Acute glomerulonephritis
Acute cystitis
Nephrolithiasis
A 55-year-old man comes to the emergency department with pain on urination, fever and chills. He
also complains of perineal and suprapubic tenderness as well as dysuria and hesitancy. His allergies
include codeine, sulfonamides, and quinidine. Temperature is 38.5 C (101.3 F), blood pressure is
132/90 mm Hg, pulse is 88/min, and respirations are 18/min. Abdominal examination is remarkable
for suprapubic tenderness. Digital rectal examination demonstrates a swollen, boggy, and exquisitely
painful prostate gland. Laboratory studies show a leukocyte count of 11,500/mm3, creatinine of 0.9
mg/dL, and blood urea nitrogen of 16 mg/dL. A urinalysis shows too numerous to count white blood
cells and Gram-negative rods. The most appropriate treatment for this patient is
amoxicillin/clavulanate 875 mg by mouth twice daily for 14 days
ceftriaxone 1 gram intravenously daily for 5 days
ciprofloxacin 500 mg by mouth twice daily for 14 days
D.
E.
78.
A.
B.
C.
D.
E. *
79.
A.
B. *
C.
D.
E.
80.
A. *
B.
C.
D.
E.
81.
A.
B.
C.
D. *
E.
82.
clindamycin 300 mg 4 times daily for 10 days
trimethoprim-sulfamethoxazole 1 double strength tablet twice daily for 14 days
A 30-year-old woman comes to the clinic complaining of loss of control of urination and "dribbling"
of urine. She has had recurrent urinary tract infections over the past 11 months, which were treated
with antibiotics. She has been following safe sex practices and denies any history of sexually
transmitted diseases, but complains of moderate pain during intercourse. Physical examination and
pelvic examination are normal. The most appropriate next investigation in this patient is
CT scan of the abdomen and pelvis
laparoscopy
pelvic sonography
renal sonography
voiding cystourethrogram
A 78-year-old woman is admitted to the hospital because of fever, flank pain, and mental
obtundation. On arrival to the hospital, she was minimally responsive and was found to have a white
blood cell count of 43,000/mm3 with a profound left shift. Urinalysis revealed packed white cells. A
renal ultrasound demonstrated a left hydronephrosis and hydroureter. A CT scan confirmed the
presence of an obstructing stone. Her vital signs on admission showed a temperature of 39.5 C (103.1
F), blood pressure of 80/40 mm Hg, pulse of 112/min, and respirations of 18/min. Intravenous
pressors were initiated. Three sets of blood cultures came back positive for Gram-negative rods
within 2 hours. The most appropriate next step in management is to
administer antibiotics, intravenously, and observe
insert a nephrostomy tube, percutaneously
perform extracorporeal shock wave lithotripsy (ESWL)
schedule cystoscopy and ureteral stent placement
schedule a nephrectomy
A 37-year-old comes to the clinic for a required pre-employment physical examination. She has no
past medical history and has no complaints. Her temperature is 37.0 C (98.6 F), blood pressure is
110/60 mm Hg, pulse is 63/min, and respirations are 14/min. She is currently menstruating. A urine
culture, which is required by her new job, reveals greater than 100,000 E. Coli colony-forming units.
The most appropriate next step in management of this patient's urine culture findings is
no further management is indicated
prescribe ciprofloxacin, orally, for 14 days
prescribe trimethoprim-sulfamethoxazole, orally, for 3 days
repeat urine culture in 2 weeks
send her for urological evaluation
A 31-year-old woman presents to your office complaining of flank pain and fever for the past 2 days.
She has a history of multiple urinary tract infections. She has suffered 2 in the previous 18 months
that were treated successfully with trimethoprim-sulfamethoxazole. She is sexually active and her last
menstrual period was 6 days ago. She takes no routine medications. Her temperature is 39.5 C (103.1
F), blood pressure is 130/65 mm Hg, and pulse is 110/min. She has prominent right costovertebral
angle (CVA) tenderness. The most appropriate management of this patient is to
admit her to the intensive care unit
initiate therapy with ampicillin and gentamicin
obtain blood cultures and send her home pending results
obtain urine cultures and initiate therapy with ampicillin and gentamicin
obtain urine cultures and send her home pending results
A 27-year-old college freshman complains of dysuria and urinary frequency. Urinalysis reveals 8to
10 WBCs per high-power field and numerousgram-negative bacteria. All of the following statements
concerning this disease are true EXCEPT that
A.
B.
C.
D. *
E.
83.
A.
B.
C. *
D.
E.
84.
A.
B. *
C.
D.
E.
85.
A. *
B.
C.
D.
E.
86.
A.
B.
C.
D. *
E.
87.
A.
B.
C.
a single dose of an antibiotic may be sufficient treatment
pregnant women with bacteriuria should be treated, even if asymptomatic
patients with flank pain or fever should be treated for 10 to 14 days
patients with indwelling catheters should receive long-term suppression
radiological investigation is appropriate formales of any age after their first infection
A 72-year-old man has the sudden onset of suprapubic pain and oliguria. His temperature is 38.0°C
(100.4°F), pulse is 100/minute, respirations are 12/minute, and BP is 110/72 mm Hg. Abdominal
examination is only re_markable for a tender, distended urinary bladder.Immediate management of
this patient should be to
obtain a flat plate of the abdomen
perform abdominal ultrasonography
insert a urethral catheter
administer furosemide IV
obtain an IVP
A man, aged 25, presents with facial edema, moderate back pains, body temperature of 37,5 С, BP
180/100 mmHg, hematuria [ up to 100 in v/f], proteinuria [2,0 g/L], hyaline casts - 10 in v/f., specific
gravity -1020. The onset of the disease is probably connected with acute tonsillitis 2 weeks ago. The
most likely diagnosis is:
Acute pyelonephritis
Acute glomerulonephritis
Cancer of the kidney
Urolithiasis
Chronic glomerulonephritis
A 29-year-old woman is critically ill. The illness was manifested by oliguria, edema, severe
headache. Pasternatsky’s sigh is negative in both sides. On lab exam, WBC of 10.000/mcL; on
urinalysis protein of 1,6g/L, erythrocyturia. Your preliminary diagnosis.
Acute pyelonephritis
Acute glomerulonephritis
Cancer of the kidney
Urolithiasis
Chronic glomerulonephritis
A 57-year-old man undergoes intravenous pyelography (IVP) as part of a workup for hypertension. A
3-cm solitary radiolucent mass is noted in the left kidney; the study otherwise is normal. The man
complains of no symptoms referable to the urinary tract, and examination of urinary sediment is
within normal limits. Which of the following studies should be performed next?
Repeat intravenous pyelography in 6 months
Early-morning urine collections for cytology (three samples)
Selective renal arteriography
Renal ultrasonography
CT scanning (with contrast enhancement) of the left kidney
A 24- year-old patient with a history of preceding streptococcal infection complains of malaise,
headache, anorexia, subfebrile fever. On exam, mild generalized edema, BP of 150/90 mm Hg, HR of
100/min, RR of 20/min, the urinalysis showed increased protein, red cell casts and hyaline casts.
What is your diagnosis?
Bacterial endocarditis
Essential hypertension
Rheumatic fever
D.
E. *
88.
A.
B. *
C.
D.
E.
89.
A.
B.
C. *
D.
E.
90.
A. *
B.
C.
D.
E.
91.
A.
B.
C. *
D.
E.
92.
A.
B.
C.
D.
E. *
93.
A. *
B.
Acute pyelonephritis
Acute glomerulonephritis
A 31-year-old woman is critically ill. The illness was manifested by oliguria, edema, severe
headache. Pasternatsky’s sigh is negative in both sides. On lab exam, WBC of 10.000/mcL; on
urinalysis protein of 1,6g/L, erythrocyturia. Your preliminary diagnosis.
Acute pyelonephritis
Acute glomerulonephritis
Urolithiasis
Cancer of the kidney
Chronic glomerulonephritis
Which area is more vulnerable to ischemic damage?
Subepicardium
Midmyocardium
Subendocardium
Pericardium
Endocardium
A 40-year-old man complained of headache in occipital area. On physical examination, the skin was
pale; there was face and hand edema, blood pressure of 170/130 mm Hg. On EchoCG, there was
concentric hypertrophy of the left ventricle. Ultrasound examination of the kidneys revealed thinned
cortical layer. Urinalysis showed proteinuria of 3.5 g/day. What is the diagnosis?
Chronic glomerulonephritis.
Essential arterial hypertension.
Chronic pyelonephritis.
Polycystic disease of the kidneys.
Chronic renal failure
A 19 year-old guy fell ill with acute nephritis after exposure. 2 weeks ago he suffered a sore throat.
Indicate the most likely etiology of nephritis.
Aurococcus
Pseudomonas aeruginosa
hemolytic streptococcus
Candida
Colon bacillus
A patient A, aged 22, was diagnosed with chronic renal failure. The blood serum showed reductions
in the amount of calcium, increasing of alkaline phosphatise degree, and the radiograph of the
forearm bones and wrist confirmed signs of osteoporosis. Which of these drugs should be applied for
treatment of the patient’s osteopathy?
multivitamins and manganese
Calcium without vitamin D
Phosphorus and Vitamin B1
Zinc and molybdenum
Vitamin D and calcium
A patient on the third day after the beginning of an acute respiratory infection complaints low back
pain, nausea, dysuria, oligouriya appeared. In urine there was hematuria. The blood creatinine 0.20
mmol / L, potassium - 6.5 mmol / liter. The urine: the specific gravity 1003, the red blood cells
(leached) - 150-200 in the field of view. What is the diagnosis?
tubulointerstitial nephritis
Acute glomerulonephritis
C.
D.
E.
94.
A.
B. *
C.
D.
E.
95.
A.
B. *
C.
D.
E.
96.
A. *
B.
C.
D.
E.
97.
A.
B. *
C.
D.
acute renal failure
Acute cystitis
Acute renal colic
A 18 year old guy gets ampicillin for treatment of acute respiratory illness complicated by acute
bronchitis. On the 5th day of treatment symptoms of intoxication increased, mild oedema on face,
pain in the joints appeard. On the skin of the trunk papular rash came out. Blood pressure: 140/90
mm Hg Throughout the day he singled out 210 ml of urine (fluid was 2000 ml). The blood test:
creatinine 0.24 mmol / l, urea 11.8 mmol / L, potassium 3.8 mmol / l, sodium 125 mmol / liter. The
urine test: proteinuria 0.99 g / l, erythrocyturia, eosinophilic leukocyturia. The relative density of
urine during the day ranged from 1002 to 1010. Urine culture for sterility gave no growth. What is
the most likely diagnosis?
Acute pyelonephritis
Acute interstitial nephritis
Acute glomerulonephritis with nephritic syndrome
Acute glomerulonephritis with nephrotic syndrome
Dysmetabolic nephropathy
The 22 year old patient during 5 months was worried about trump pain, low-grade fever, frequent
urination. The urine test: moderate proteinuria, white blood cells in the entire field of view, the
bacteriuria. The blood - leukocytosis, erythrocyte sedimentation rate increasing. Which of the
following diagnoses is most likely?
Renal amyloidosis.
Chronic pyelonephritis.
Acute glomerulonephritis.
Acute pyelonephritis.
Chronic glomerulonephritis.
Patient K., 24 years old, is cured in the intensive care unit from a long compression syndrome, acute
renal failure. The laboratory examination: urea 28.4 mmol / l, creatinine 820 micromol / l, potassium
8.4 mmol / l, hemoglobin 76 g / l, hematocrit 0.26 l / l, erythrocytes 2.4 T / L. Which complications
can directly threaten the patient's life?
hyperkalemic cardiac arrest
Anemia
Respiratory distress syndrome
DIC
hemic hypoxia
Patient M., 19 years old, was taken to hospital in serious condition with complaints of general
weakness, thirst, dry mouth, dull headaches. The first time he felt himself bad a year ago, when blood
pressure began to rise (180/110 mmHg. Art..) Treatment was not regular. His well-being became
worse a few days ago. During examination: face puffiness, pale skin and mucous membranes, muscle
twitching. Tongue was covered with gray bloom, bleeding gums. There was mild oedema on the
shins. Pulse is 94 in 1 minute, rhythmic, tense. BP 175/120 mm Hg. Art. The first tone over the top of
the heart is weakened; the second is stronger above the aorta. In the lower parts of lungs breathing is
hard. The liver acts 2 cm from the edge of the rib arch, it is a little condensed, sensitive during
palpation. Blood tests: er. - 2.2 h1012 / l Hb - 82 g / l, ESR-26 mm / hour. Urine test: some density
1.016 protein, 1.35 g / l, leukocytes 6 n / s, erythrocytes. - densely cover the field of vision, single
hyaline cylinders. What is your diagnosis?
Pyelonephritis
Glomerulonephritis
Nephrolithiasis
Focal nephritis
E.
Amyloidosis
98.
Patient D, aged 20, complains about headaches, dizziness, general weakness, nausea and vomiting
after every meal. One year ago, it was accidentally discovered increased blood pressure. Further was
neither observed nor treated. He felt worse a week ago. 10 years ago, changes in urine test were
found, but he was not examined or treated. During examination: the patient feels malnutrition, is of
serious condition, pale, and has puffy face. Tongue is dry, covered with gray patina. Gums are
bleeding. Pulse counts 84 beats per minute, is tight, rhythmic, blood pressure is 180/110 mm Hg
Cardiac impulse is spilled, the left border of the heart is displaced to the left for 2 cm. The first tone
above the top of the heart is weakened, systolic murmur over the top is heard, the second tone is
stronger above the aorta. In the lower parts of lungs breathing is hard, abdomen is soft, the liver acts
2 cm from the edge of the rib arch, it is dense, sensitive during palpation. On the skin of the trunk
there are traces of scratching. What is your preliminary diagnosis?
Multiple myeloma
Polycystic
Pyelonephritis
Primary chronic glomerulonephritis. CRF V st.
Nephrolithiasis
A 24 year old patient was diagnosed with acute glomerulonephritis. Urine test: specific gravity 1010,
Protein - 1.65 g / l, erythrocytes. 5-7 p / s, lakes. 2-3 p / h. blood Creatinine - 0.587 mmol / liter.
What is the main reason of hyperazotemia which suffers the patient with acute glomerulonephritis?
Reduced tubular reabsorption.
The decrease in renal blood flow.
Strengthening the collapse of erythrocytes and leukocytes in the glomerular filtrate.
Reduced glomerular filtration rate.
Violation of protein metabolism.
A 34 year old patient, 3 months ago began to suffer from headache, shortness of breath when
walking, loss of appetite, nausea and weakness. He has been suffering from chronic
glomerulonephritis for10 years. It was suspected that during glomerulonephritis chronic renal
insufficiency was complicated. Which laboratory parameters will be the most informative in this
case? creatinine 0.46 mmol / l
Blood
Blood urea 7.3 mmol / liter.
Blood sodium 130 mmol / liter.
Urea nitrogen 20 - 40% of the residual nitrogen.
Potassium blood 5.2 mmol / liter.
A doctor discovered high blood pressure in a 19 year old patient. Urine test: leukocytes - 6-7,
erythrocytes - 3-4 in n / view, protein 0.066 g / litre. Which instrumental examination should be
appointed for clarification of the kidneys’ state?
Ultrasound of the kidneys
radioisotope renography
CT
Survey radiography of the kidneys
renal angiography
The 25-year-old patient with rheumatoid arthritis of joint and visceral form blood pressure and
proteinuria have been increasing during the last six months. Development of renal amyloidosis was
suspected. Necessary examinations are:
Kidney biopsy, biopsy of the rectal mucosa, a biopsy of the gums
Definition of haematuria, renal biopsy
Determination of proteinuria, kaliyuria, uraturia
A.
B.
C.
D. *
E.
99.
A.
B.
C.
D. *
E.
100.
A. *
B.
C.
D.
E.
101.
A. *
B.
C.
D.
E.
102.
A. *
B.
C.
D.
E.
103.
A.
B. *
C.
D.
E.
104.
A. *
B.
C.
D.
E.
105.
A.
B.
C. *
D.
E.
Determination of proteinuria, renal biopsy
Biopsy of intestinal mucosa, the definition of pyuria
During the examination at clinic of 23 year old girl acute glomerulonephritis, nephrotic syndrome
without renal dysfunction were defined. Assign the leading drug of etiological treatment for the
patient.
Prednisolone
Augmentin
Biseptol
Heparin
saluretic
A 63-year-old man is admitted with acute somnolence, disorientation, and right upper and lower
extremity weakness. He has a 3-year history of renal insufficiency and congestive heart failure
attributed to long-standing hypertension, which has been poorly controlled in part because of poor
adherence. On physical examination, the blood pressure is 160/96 mm Hg, pulse rate 11 0/min,
respiratory rate 14/min, and temperature 38 °C (100.4 °F). There is three-fingerbreadth neck vein
distention while sitting and hepatojugular reflux. Cardiac examination shows an S3 gallop;
pulmonary examination reveals bilateral crackles. The abdomen is benign, and 2+ lower extremity
edema is present. Right biceps and patellar reflexes are increased, and a right Babinski response is
noted. Laboratory studies: Hematocrit - 33%. Leukocyte count - 10,700/?L. Platelet count- Normal
Blood urea nitrogen - 35 mg/dL. Serum creatinine - 2.3 mg/dL Serum sodium - 128 meq/L. Serum
potassium - 4.3 meq/L. Serum chloride - 93 meq/L. Serum bicarbonate - 16 meq/L. Arterial blood
gas pH 7.30, PCO2 - 33 mm Hg. Urinalysis pH 6.0, specific gravity 1.014, 2+ proteinuria, no
hematuria or ketonuria; no formed elements on microscopy Noncontrast computed tomography of the
head shows only an ill-defined mass effect. Neurology and neurosurgical consultants request contrast
computed tomography to more precisely demarcate the suspected tumor and thus pinpoint a site for
biopsy or resection. What is the most important next step to prepare this patient for a contrast study?
Administer acetylcysteine
Administer half-normal saline
Administer dopamine
Administer bicarbonate
Administer calcium channel blocker
A 70-year-old woman is admitted because she has had malaise and anorexia for 1 week. She has been
previously healthy, except for hypertension and hypercholesterolemia, treated with
hydrochlorothiazide and atorvastatin. On physical examination, the supine blood pressure is 150/95
mm Hg, pulse rate 80/min, respiratory rate 20/min, and temperature 37.4 °C (99.3 °F). The blood
pressure is 125/80 mm Hg and the pulse rate 96/min while standing. There is no neck vein distention
or hepatojugular reflux. Cardiac, breast, abdominal, and pulmonary examinations are normal. No
lower extremity edema is present. Laboratory studies: Hematocrit 29% Leukocyte count 3,200/?L
Platelet count - 90,000/?L. Blood urea nitrogen - 62 mg/dL. Serum creatinine - 4.6 mg/dL. Serum
sodium - 134 meq/L. Serum potassium - 5.0 meq/L. Serum chloride - 114 meq/L. Serum bicarbonate
- 15 meq/L. Serum glucose - 105 mg/dL. Serum calcium - 12.5. mg/dL. Serum inorganic phosphate
8.5 mg/dL. Urine creatinine - 25 mg/dL. Urine sodium - 50 meq/L. Urinalysis Specific gravity 1
.007; trace proteinuria; no glucosuria or ketonuria Arterial blood gaspH 7.30, PCO2 28 mm Hg.
Microscopic analysis shows scattered tubular epithelial cells. Posteroanterior and lateral films of the
chest are normal. What is the most likely diagnosis?
Milk-alkali syndrome
Sarcoidosis
Multiple myeloma
Primary hyperparathyroidism
Clinical consequence of hydrochlorothiazide therapy
106.
A.
B.
C.
D. *
E.
107.
A. *
B.
C.
D.
E.
108.
A.
B.
C. *
D.
E.
28-year-old man has the acute onset of colicky pain in the left costovertebral angle radiating into the
groin, as well as gross hematuria. Abdominal x-ray discloses a stone in the left ureter. All of the
following are true statements concerning this disease EXCEPT that
the majority of renal stones are radiopaque
radiolucent stones are usually composed of uric acid
staghorn calculi are associated with alkaline urine
radiopaque stones usually contain cystine
urate stones are associated with acidic urine
A 69-year-old white man is referred for worsening hypertension over the past 2 to 3 months. He has
had hypertension for the past 18 months. It had been controlled by ?-blocker therapy, which was
begun after he had an inferior myocardial infarction. A recent blood pressure measurement was
200/120mm Hg, requiring additional therapy with amlodipine. His medical history is significant for
the myocardial infarction and a right femoral popliteal bypass. He smokes two packs of cigarettes
daily and drinks alcohol socially. On examination, blood pressure is 178/104 mm Hg seated and
standing, and body weight is 72 kg (159 Ib). Optic funduscopy reveals background hypertensive
retinopathy. A left carotid bruit is heard. Cardiopulmonary and neuromuscular examinations are
normal. Abdominal examination showed no organomegaly, but an epigastric bruit is present. No
peripheral edema is noted. Serum creatinine concentration is 2.3 meq/dL, and serum potassium
concentration is 3.9 mg/dL. Urinalysis shows 1 + proteinuria without hematuria. Electrocardiography
is positive for left ventricular hype rtrophy. What is the most appropriate noninvasive screening test
for possible renal artery stenosis in this patient?
Magnetic resonance angiography with gadolinium
Computed tomographic angiography with contrast
Captopril renography
Captopril plasma renin activity test
none of them
A 61-year-old woman with a previously normal serum creatinine concentration is admitted with
abdominal pain. Abdominal aortic aneurysm was diagnosed after an intravenous contrast study, and
the patient underwent emergency aneurysmectomy. On the third hospital day, in the intensive care
unit, the patient was oliguric. She was given an intravenous furosemide infusion but was still oliguric
several hours later. On physical examination, the blood pressure is 90/62 mm Hg, with no orthostatic
changes; pulse rate 11 5/min; respiratory rate 22/min; and temperature 36.8 °C (98.2 °F).
Three-fingerbreadth neck-vein distention at 45 degrees and hepatojugular reflux are present. Cardiac
examination shows an S3 gallop. There are scattered bibasilar rales. The abdomen has a fresh
surgical scar. Bowel sounds are not heard, and there is diffuse tenderness. The patient has 2+ lower
extremity edema. She is arousable but somnolent and moves all extremities in response to
commands. She complains of dyspnea. Laboratory studies: Hematocrit - 37% Leukocyte count
Leukocytosis Platelet count - Low Blood urea nitrogen 75 mg/dL Serum creatinine - 4.4 mg/dL
Serum sodium - 130 meq/L Serum potassium - 6.3 meq/L Serum chloride - 90 meq/L Serum
bicarbonate - 16 meq/L Arterial blood gaspH 7.26, Pco2 - 25 mm Hg, Po2 - 65 mm Hg Urinalysis pH
6.0, specific gravity 1.009, 2+ proteinuria, no hematuria or ketonuria There are muddy brown casts
on microscopic examination. The electrocardiogram shows prominent, peaked T waves. What is the
Administer
next step in fenoldopam
treatment of this patient with acute renal failure?
Start ultrafiltration
Start continuous venovenous hemofiltration
Start peritoneal dialysis
Initiate plasma exchange
109.
A 52-year-old man is referred by his primary care physician for hypertension and hypokalemia over
the past 6 months. Blood pressure and routine chemistries were normal last year at the time of an
executive physical. He has no history of cardiovascular disease, stroke, or renal disease. Family
history is negative for hypertension. He uses alcohol socially and does not smoke but chews tobacco.
He takes no medications regularly. On examination, the patientweighs 77kg (168 Ib). Blood pressure
is 164/102mm Hg seated and standing. Except for trace pedal edema, the remainder of examination is
normal. The primary care physician provides the following laboratory values: Blood urea nitrogen 21 mg/dL Serum creatinine - 0.9 mg/dL Serum sodium - 141 meq/L Serum potassium
3.1 meq/L
Serum chloride
100 meq/L Serum bicarbonate
28 meq/L A 24-hour urine test during salt
loading reveals the following values: Creatinine - 1.1 g Sodium - 252 meq Potassium 128 meq The
daily aldosterone excretion rate is 6 mg (normal, 5 to 15 mg), plasma renin activity is 1 ?g/L/h, and
plasma aldosterone level is 9 ng/dL. Which diagnostic test would you order next?
A.
B. *
Computed tomography of the adrenal glands
Serum cortisol and urinary free cortisol measurement
Magnetic resonance angiography with gadolinium
Adrenocorticotropin hormone stimulation test
none of them
A 64-year-old black woman has had hypertension for 25 years. On therapy, her blood pressure has
been in the range of 140 to 160/95 to 100 mm Hg. She presents for blood pressure management.
Review of past laboratory data shows that the serum creatinine concentration was 1 .2 mg/dL 10
years ago, 1.7 mg/dL 5 years ago, 2.0 mg/dL 1 year ago, and 1.9 mg/dL 2 months ago. Recent
urinalysis shows 2+ proteinuria, no hematuria, and occasional granular and hyaline casts. Urine
protein:creatinine ratio is 0.5. Renal ultrasonography shows no hydronephrosis with kidney sizes at
9.5 cm bilaterally. On examination, body weight is 84 kg (185.2 Ib) and temperature is 36.9 °C
(98.4°F). Blood pressure is 148/96 mm Hg in both arms. Optic funduscopy shows moderate arteriolar
sclerosis and constriction. No jugulovenous distention is present. The lungs are clear. Cardiac
examination shows regular sinus rhythm, S4 but no S3, and no murmur. There is no edema of the
extremities. What is the most likely cause of this patient’s renal disease?
Membranous glomerulopathy
Obstructive uropathy
IgA nephritis
Nephrosclerosis
Obesity-related glomerular disease
Diabetic nephropathy results from an elevation of blood pressure, increasing the workload of the
glomeruli. The glomeruli thicken and allow serum albumin to pass into the urine. Which of the
following signs and symptoms indicate diabetic nephropathy?
Edema around the eyes upon awakening, progressing to general swelling of the legs and body
Weight gain, malaise, fatigue, and frothy urine
Rust-colored urine, weight loss, and back pain
A and C
A and B
A patient is admitted with the following signs and symptoms: edema around the eyes upon
awakening, progressing to general swelling of the legs and body; weight gain; fatigue; headache;
nausea; vomiting; frequent hiccoughs; and itching. A urine specimen is obtained, and the urine is
frothy. The patient has a history of insulin-dependent diabetes (type 1 diabetes), which is poorly
controlled, and high blood pressure, and his blood work comes back showing an elevated cholesterol
level. Which of the following disease processes would you suspect?
Nephrosclerosis
Diabetic nephropathy
Polycystic kidney disease
C.
D.
E.
110.
A.
B.
C.
D. *
E.
111.
A.
B.
C.
D.
E. *
112.
A.
B. *
C.
D.
E.
113.
A.
B.
C. *
D.
E.
114.
A.
B.
C. *
D.
E.
115.
A.
B. *
C.
D.
E.
116.
A.
B.
C. *
D.
E.
Amyloidosis
None of them
A 65-year-old woman with stage 4 chronic kidney disease secondary to hypertension comes to the
office because she has had dysuria and increased urinary frequency during the past two days. She has
not had fever or hematuria. Temperature is 37.3°C (99.1°F), pulse rate is 90/min, and blood pressure
is 142/85 mmHg. On physical examination, no flank tenderness is noted. Urinalysis shows cloudy
urine that is positive for leukocyte esterase, nitrites, and protein. Results of urine culture are pending.
Which of the following medications is the most appropriate therapy for this patient's condition?
Intravenous administration of gentamicin
Intravenous administration of vancomycin
Oral administration of ciprofloxacin
Oral administration of nitrofurantoin
Intramuscular administration of ceftriaxone
A 36-year-old woman with diabetic nephropathy comes to the office for follow-up. During the
interview, the patient says she has had fatigue for the past month. Physical examination shows pallor
and pale conjunctivae, a grade 2/6 systolic murmur that is heard best over the left lower sternal
border, and bilateral mild, pitting edema of the lower extremities. Laboratory studies show
hemoglobin level of 9.8 g/dL and estimated glomerular filtration rate, calculated using the
Modification of Diet in Renal Disease (MDRD) Study equation, is 19 mL/min/1.73 m2. Therapy with
an erythropoiesisstimulating agent is initiated. Which of the following is the target hemoglobin level
7-8
g/dLpatient?
for this
9-10 g/dL
11-12 g/dL
13-14 g/dL
15-16 g/dL
A 54-year-old woman with diabetic nephropathy comes to the office because she has had vomiting,
anorexia, fatigue, itching, and a metallic taste in her mouth for the past two weeks. Respirations are
32/min. Physical examination shows urine-like odor of the breath and 2+ pitting edema. Estimated
glomerular filtration rate, calculated using the Modification of Diet in Renal Disease (MDRD) Study
equation, is 14 mL/min/1.73 m2. Additional studies in this patient are most likely to show presence
of which of the following conditions?
Hypokalemia
Primary metabolic acidosis
Primary metabolic alkalosis
Primary respiratory acidosis
Primary respiratory alkalosis
A 67-year-old man with stage 4 chronic kidney disease, hypertension, type 2 diabetes mellitus, and
anemia of chronic kidney disease comes to the office for routine follow-up. Laboratory findings
include the following: Ca – 9,4 mg/dL, creatinine 3,24 mg/dL, BUN – 36 mg/dL. Estimation of
glomerular filtration rate, calculated using the Modification of Diet in Renal Disease (MDRD) Study
equation, is 19 mL/min/1.73 m2. Measurement of which of the following additional laboratory values
is most appropriate to screen for renal osteodystrophy in this patient?
Erythrocyte sedimentation rate
Serum C-reactive protein level
Serum intact parathyroid hormone level
Serum thyroid-stimulating hormone level
Serum vitamin B12 level
117.
A.
B.
C.
D. *
E.
118.
A. *
B.
C.
D.
E.
119.
A.
B.
C.
D.
E. *
120.
A.
B.
C.
D.
E. *
A 56-year-old woman comes to the emergency department because she has had increasing swelling
of the right ankle over the past two days, since she sustained an injury while playing outdoors with
her grandchildren. She says she has been taking over ibuprofen 400 to 800 mg every four to six hours
to relieve the pain. Medical history includes mild hypertension, which is currently controlled with
lisinopril. Results of laboratory studies show elevated levels of serum creatinine and blood urea
nitrogen. Acute renal failure induced by use of nonsteroidal anti-inflammatory drugs is suspected. If
this suspected diagnosis is correct, which of the following additional abnormal laboratory results is
most likely?
Decreased serum chloride level
Decreased serum potassium level
Decreased serum sodium level
Elevated serum potassium level
Elevated serum sodium level
A 67-year-old man with mild dementia, in whom peritoneal dialysis was recently initiated because of
end-stage renal disease, is brought to the office by his wife because he has had abdominal pain for the
past 48 hours. The patient's wife says he has been reusing one cap to close off his peritoneal dialysis
catheter. On analysis, a sample of fluid from the catheter is cloudy and grows gram-positive cocci.
Which of the following is the most appropriate next step?
Intraperitoneal administration of antibiotics
Intravenous administration of antibiotics
Retraining of the patient regarding catheter procedures
Retraining of the patient's wife regarding catheter procedures
Transfer of the patient to a hemodialysis clinic
A 60-year-old woman with end-stage renal disease secondary to diabetic nephropathy comes to the
clinic for routine hemodialysis. The patient says she has cramping during hemodialysis as well as
weakness after each treatment. Post-treatment, pulse rate is 95/min and blood pressure is 90/60
mmHg. On physical examination, auscultation of the chest shows normal breath sounds. Examination
of the extremities shows no edema. Which of the following is the most appropriate next step?
Add sodium modeling
Continue monitoring blood pressure
Decrease the dialysate temperature
Decrease the dialysis time
Increase the dry weight
A 38-year-old woman with end-stage renal disease secondary to hypertension comes to the clinic for
routine hemodialysis treatment. Blood pressure prior to the treatment is 178/100 mmHg, and the
patient has a headache. Blood pressure at the end of the treatment is 150/90 mmHg. Average pulse
rate during hemodialysis treatment is 48/min. Decreasing the patient's target weight by 4.4 lb (2 kg)
relieves the headache, but blood pressure remains increased and severe leg cramps develop. Which of
the following is the most appropriate next step?
Continue decreasing the target weight
Initiate hydralazine therapy
Initiate metoprolol therapy
Initiate sodium modeling
Initiate ultrafiltration profiling
121.
A.
B.
C. *
D.
E.
122.
A.
B. *
C.
D.
E.
123.
A. *
B.
C.
D.
E.
124.
A.
B.
A 62-year-old man comes to the office for routine follow-up. The patient has retired from civil
service with full medical benefits. Medical history includes hypertension, hyperlipidemia, coronary
artery disease, stage 3 chronic kidney disease, and type 2 diabetes mellitus. Current medications
include carvedilol, amlodipine, furosemide, lisinopril, nitroglycerin transdermal patch, insulin,
simvastatin, aspirin, and gabapentin. The patient appears well developed and well nourished, and he
is not in acute distress. Weight is 178 lb, and body mass index is 27.9 kg/m2. Pulse rate is 88/min,
and blood pressure is 172/88 mmHg in the left arm and 170/78 mmHg in the right arm. Which of the
following findings in this patient is the most likely cause of continued uncontrolled hypertension?
Body mass index
History of hyperlipidemia
Noncompliance with the drug regimen
Patient age
Polypharmacy
A 32-year-old man is referred to the office by his primary care provider for initial consultation
regarding stage 3 chronic kidney disease. The patient initially sought treatment at the primary care
office because he had pain in his lower back. An incidental finding of increased blood pressure was
noted at that time. Results of laboratory studies performed at the primary care office included
glomerular filtration rate, calculated using the Modification of Diet in Renal Disease (MDRD) Study
equation, of 59 mL/min/1.73 m2. The patient currently is not taking any antihypertensive
medications. Before further evaluation regarding the cause of hypertension and decreased glomerular
filtration rate, initiation of antihypertensive therapy is planned. Which of the following classes of
medications is the most appropriate initial antihypertensive therapy for this patient?
Alpha2-agonists
Angiotensin-converting enzyme inhibitors
Beta-blockers
Calcium channel blockers
Vasodilators
A 55-year-old man with stage 3 chronic kidney disease comes to the office for follow-up. Glomerular
filtration rate, calculated using the Modification of Diet in Renal Disease (MDRD) Study equation,
has been consistent at 39 mL/min/1.73 m2, and serum creatinine level has been steady at 1.9 mg/dL.
Medical history includes long-standing hypertension and type 2 diabetes mellitus. Current
medications include metformin 500 mg twice daily, celecoxib 200 mg daily, nifedipine 30 mg daily,
and quinapril 10 mg daily. The patient appears well developed and well nourished. Temperature is
36.4°C (97.5°F), pulse rate is 76/min, respirations are 18/min, and blood pressure is 130/78 mmHg.
On physical examination, no abnormalities are noted. Ultrasonography of the kidneys shows bilateral
renal cortical atrophy. Discontinuation of which of the following medications in this patient's
regimen is most appropriate?
Metformin and celecoxib
Nifedipine and celecoxib
Nifedipine and metformin
Nifedipine and quinapril
Quinapril and metformin
A 36-year-old man comes to the office for consultation regarding results of recent laboratory studies.
Medical history includes stage 3 chronic kidney disease and hypertension, which is currently well
controlled with lisinopril and furosemide. The patient has a 20–pack-year history of cigarette
smoking. Results of laboratory studies of serum include the following: total cholesterol 236 mg/dL,
LDL-C-128 mg/dL, HDL-C 38 mg/dL, triglycerides – 110 mg/dL. Initiation of therapy with which of
the following agents is most appropriate for management of hyperlipidemia in this patient?
Ezetimibe
Fenofibrate
C.
D.
E. *
125.
A.
B.
C.
D.
E. *
126.
A.
B.
C.
D. *
E.
127.
A. *
B.
C.
D.
E.
Fish oil
Niacin
Simvastatin
A 43-year-old man who has had recurrent sinusitis during the past year comes to the emergency
department because he has had shortness of breath and cough productive of green sputum for the past
two weeks. Physical examination shows tenderness over the maxillary region. On auscultation of the
chest, crackles are noted bilaterally in the lung bases. Trace edema of the extremities is noted. Chest
x-ray study shows bilateral pleural effusions with apical opacities. Laboratory studies show serum
creatinine level of 3.0 mg/dL and blood urea nitrogen level of 60 mg/dL. Result of antiglomerular
basement membrane antibody test is negative and result of antineutrophil cytoplasmic antibody test is
positive. Urinalysis shows pH of 5.5, specific gravity of 1.020, 2+ protein, and 2+ blood. Red blood
cells are visible in the urine sediment. Which of the following is the most likely diagnosis?
Allergic interstitial nephritis
Goodpasture syndrome
IgA nephropathy
Poststreptococcal glomerulonephritis
Wegener granulomatosis
A 35-year-old man with no past medical history presents to the emergency department (ED) with
cough and shortness of breath that have been present for 1 week. He reports occasional blood-tinged
sputum. He denies malaise, weight loss, fevers, or joint pain. Blood pressure is 140/80 mm Hg, heart
rate is 80 bpm, and oxygen saturation is 97% on room air. Laboratory testing reveals a blood urea
nitrogen (BUN) level of 48 mg/dL (normal, 11-23 mg/dL) and serum creatinine level of 3.5 mg/dL
(normal, 0.6-1.2 mg/dL). Serum electrolytes are normal. Urinalysis reveals 1+ protein, moderate
blood, 3 to 4 red blood cell casts, and 2 to 3 granular casts per high-power field. Chest radiograph
shows infiltrates in both lungs. A spot urine protein/creatinine ratio is 0.6 g/mg. Enzyme-linked
immunoassay for anti-glomerular basement membrane (GBM) antibodies is positive. A titer for
antineutrophil cytoplasmic antibodies (ANCA) is undetectable. The patient undergoes a renal biopsy.
Light microscopy reveals crescents in the glomeruli, and immunofluorescence reveals linear staining
of IgG along the glomerular capillaries. What is the best management strategy for this patient?
Dialysis
Intravenous (IV) prednisolone
Observation
Plasma exchange, prednisone, and cyclophosphamide
None of them
A 9-year-old boy presents to the ED with his mother with puffy eyes and scant urine. His general
health has been good until 2 weeks ago when he developed a sore throat and swollen glands. An
antibiotic was started at that time. He appears well except for facial swelling and edema in his feet.
Blood pressure is 150/90 mm Hg. The remainder of the physical examination is normal. Laboratory
testing reveals a serum creatinine level of 1.8 mg/dL, BUN of 35 mg/dL, albumin level of 3.2 g/dL
(normal, 3.3-5.2 g/dL), serum complement C3 level of 80 mg/dL (normal, 100-233 mg/dL), serum
complement C4 level of 25 mg/dL (normal, 14-18 mg/dL), anti-streptolysin O titer of 230 U (normal,
< 200 U), and an antinuclear antibody titer of 1:20 (> 1:40 is abnormal). Urinalysis reveals 1+
protein, 10 to 20 red blood cells, 2 to 6 white blood cells, and occasional red blood cell casts. Urine
protein is 2 g/24 hr. What is the next step in the management of this patient?
Diuretics and antihypertensive medication
IV prednisone
Kidney biopsy
Oral cyclophosphamide
None of them
128.
A. *
B.
C.
D.
E.
129.
A. *
B.
C.
D.
E.
130.
A. *
B.
C.
D.
E.
131.
A.
A 60-year-old man presents to the ED with a 6-week history of fatigue, myalgias, weight loss, and
shortness of breath. Past medical history is significant for hypertension for 10 years. Urine output and
blood pressure are normal. There is no skin rash, hepatomegaly, splenomegaly, or peripheral edema.
Laboratory testing reveals a hemoglobin level of 10 g/dL (normal, 13-18 g/dL), BUN of 68 mg/dL,
and serum creatinine level of 4.5 mg/dL. Serum electrolytes are normal. Serum creatinine 1 year ago
was 0.9 mg/dL. Urinalysis shows 2+ protein, 15 to 20 red blood cells, 5 to 10 white blood cells, and a
few erythrocyte casts and granular casts per high-power field. Complement levels are normal.
Ultrasound of the kidneys reveals 11-cm kidneys bilaterally with no hydronephrosis. Chest
radiograph reveals patchy infiltrates in both lungs suggestive of bilateral multilobar pneumonia. The
patient received 2 L of normal saline with no improvement in serum creatinine. Results of testing for
serum anti-GBM antibodies, ANCA, and antinuclear antibody, serum protein electrophoresis, and
urine electrophoresis are pending. What is the next step in the management of this patient?
Emergent renal biopsy
Lung biopsy
Nasal and sinus biopsies
Wait for results of serologic testing and continue supportive therapy
None of them
Presents to the ED with painless swelling in both ankles and a 10-lb weight gain over the past 3
months. During a physical examination 1 year prior, 2+ protein was noted on dipstick urinalysis, but
the patient denied further evaluation because he felt well. There is no family history of renal disease.
Blood pressure is 120/80 mm Hg. Physical examination is notable for edema in his legs up to the mid
thighs. Laboratory testing reveals a hemoglobin level of 14 g/dL, hematocrit of 42%, serum glucose
level of 80 mg/dL, serum creatinine level of 1.1 mg/dL, BUN of 28 mg/dL, albumin level of 2.6
g/dL, serum total cholesterol level of 325 mg/dL (normal, < 200 mg/dL), and serum triglyceride level
of 800 mg/dL (normal, < 160 mg/dL). Serum complement levels are within normal limits. Urinalysis
demonstrates 4+ protein on dipstick. Urine microscopy reveals 0 to 2 erythrocytes, hyaline casts, oval
fat bodies, and fatty casts. A spot protein/creatinine ratio is 6 g/mg. The patient undergoes a renal
biopsy. Renal biopsy results are as suspected. A careful inquiry into the use of nonsteroidal
anti-inflammatory drugs and other drugs is negative. Serologic testing for hepatitis B and C is
negative. An age-appropriate work-up for malignancy is negative. What is the next step in this
Furosemide
and an angiotensin-converting enzyme (ACE) inhibitor
patient’s management?
Monthly albumin infusions
Oral prednisone
Warfarin
None of them
A 36-year-old Asian man with a history of asthma presents with a complaint of red urine. He
describes 5 days of nasal congestion and dry cough. He notes no sore throat, fever, chills, myalgias,
arthralgias, or flank pain. He has no family history of renal disease. A urine study indicates
1+ protein, and no bacteria, leukocyte esterase, or nitrates. Thirty to 50 erythrocytes are observed, but
no leukocytes are present. His serum creatinine level is normal. Which one of the following is the
mostnephropathy
likely diagnosis?
IgA
Nephrolithiasis
Wegener’s granulomatosis
Postinfectious glomerulonephritis
None of them
A 64-year-old Caucasian man with a history of hypertension, hyperlipidemia, and nephrolithiasis
presents with a complaint of dark-colored urine. He felt well until 2 days ago when he noted
increasing fatigue and muscle weakness. Dipstick urinalysis shows a specific gravity of 1.020, no
protein, and large blood. Neither leukocyte esterase nor nitrates are identified in the urine. The
sediment reveals no erythrocytes or leukocytes. Which of the following diagnostic procedures is
appropriate at
point?for urologic cancer
Cystoscopy
to this
evaluate
B.
C.
D. *
E.
132.
A.
B.
C.
D. *
E.
133.
A.
B.
C. *
D.
E.
134.
A. *
B.
C.
D.
E.
135.
A.
B.
C. *
D.
Flank computed tomography (CT) to evaluate for recurrent nephrolithiasis
Kidney biopsy to delineate the disease process
Serum analysis to evaluate the level of creatinine
None of them
A 61-year-old woman with hypertension, type 2 diabetes mellitus, ischemic cardiomyopathy, and
chronic renal insufficiency reports pain in her right knee. Her blood pressure is 140/84 mm Hg, and
her pulse is 70 bpm. Because of tenderness and effusion in the knee joint, the patient is prescribed
celecoxib 200 mg once daily. After 14 days of therapy, she reports dyspnea, increased swelling in the
lower extremities, and fatigue. Blood pressure is now 188/100 mm Hg, blood urea nitrogen (BUN) is
67 mg/dL (baseline, 41 mg/dL), and serum creatinine level is 3.9 mg/dL (baseline, 1.9 mg/dL).
Which of the following is the most likely mechanism by which celecoxib caused acute renal failure?
Acute papillary necrosis with renal obstruction
Acute tubular necrosis from drug-induced nephrotoxicity
Drug reaction causing allergic interstitial nephritis
Hemodynamic renal insufficiency from loss of compensatory prostaglandins induced by
cyclooxygenase-2 inhibition of celecoxib
None of them
A 31-year-old man with a 4-year history of HIV infection who takes zidovudine and lamivudine
begins receiving indinavir to further reduce the viral load. He also continues taking
trimethoprim-sulfamethoxazole 3 times weekly. Over the next 12 weeks, he develops nausea with
vomiting, anorexia, and an episode of gross hematuria. Urinalysis results show hematuria and pyuria.
Urine sediment examination shows crystals in various starburst and plate-like patterns. Serum BUN
(54 mg/dL) and serum creatinine (2.5 mg/dL) are elevated. Indinavir is discontinued, and the patient
receives an intravenous infusion of 0.9‰ saline. Which of the following most likely caused his acute
Acute
tubular necrosis caused by indinavir
renal failure?
Allergic interstitial nephritis caused by trimethoprim-sulfamethoxazole
Indinavir-associated crystal-induced renal failure
Obstructive uropathy from retroperitoneal nodes caused by HIV-associated lymphoma
None of them
A 71-year-old man with type 2 diabetes mellitus, gout, hypertension, hyperlipidemia, and chronic
renal insufficiency (serum creatinine, 2.8 mg/dL) has chest pain and electrocardiographic changes
consistent with myocardial ischemia. Prior to cardiac catheterization, he is given fluids intravenously
to reduce contrast-associated renal injury. He receives 120 mL of non-iodinated, low osmolarity
contrast during the procedure and develops transient hypotension. Over the next few days, he
develops severe hypertension, purple toes on the right foot, and gastrointestinal bleeding. His serum
creatinine level increases to 6.5 mg/dL, necessitating hemodialysis. Which of the following most
likely caused his renal failure?
Cholesterol embolization to the small arteries and arterioles in the kidney
Congestive heart failure with prerenal azotemia
Ischemic acute tubular necrosis caused by hypotension during catheterization
Radiocontrast-induced nephrotoxicity
None of them
In a patient who has developed severe hyperkalemia and associated changes on electrocardiogram (ie,
peaked T waves, widened QRS complex), administration of which of the following agents is the most
appropriate initial therapy?
A ?2-agonist, via a nebulizer
Calcium gluconate, intravenously
Insulin plus glucose, intravenously
Sodium polystyrene sulfonate, orally
E.
136.
A.
B.
C. *
D.
E.
137.
A.
B.
C. *
D.
E.
138.
A.
B.
C.
D. *
E.
139.
A.
B. *
None of them
A 28-year-old obese woman presents to her doctor’s office with persistent headache and malaise. She
is otherwise healthy. Blood pressure measured in the office is 190/110 mm Hg. Serum electrolytes
demonstrate hypokalemia (serum potassium, 3.0 mEq/L) and metabolic alkalosis (serum bicarbonate,
32 mEq/L) with normal kidney function. Urinalysis reveals no proteinuria, cylinduria, or casts. A
secondary cause of hypertension is considered. Renal ultrasound shows normal echogenicity, with
the right kidney 12.0 cm and the left kidney 10.5 cm in length. Doppler examination is technically
limited by her obesity. A renal angiogram reveals significant stenosis (90%) of the left renal artery
due to fibromuscular dysplasia. What is the best management option for this patient at this point?
Perform renal artery bypass
Serial ultrasonography to assess kidney size
Perform percutaneous angioplasty of the stenotic lesion
Start an angiotensin-converting enzyme (ACE) inhibitor and titrate to control blood pressure
None of them
A 76-year-old man with a past history of non– insulin-dependent diabetes mellitus and hypertension
is seen in the office. His blood pressure is 140/85 mm Hg, and he has mild renal insufficiency (serum
creatinine, 1.4 mg/dL). Urinalysis reveals 2+ protein on dipstick but no cylinduria. Blood pressure
medications include hydrochlorothiazide and atenolol. What antihypertensive agent will best control
blood pressure and preserve renal function in this patient?
Amlodipine
Furosemide
Lisinopril
Verapamil
None of them
A 38-year-old woman who is 36 weeks pregnant presents for routine follow-up. She has had
hypertension since age 34 years, which requires antihypertensive therapy. Prior to her pregnancy, the
patient’s blood pressures averaged 130/70 mm Hg on amlodipine. During her first trimester, her
blood pressure averaged 120/60 mm Hg but has risen in recent weeks to 150/95 mm Hg on the same
therapy. The patient’s only complaint is of worsening lower extremity edema. A 24-hour urine
collection reveals 1500 mg of protein. Serum laboratory values, including electrolytes, liver function
tests, and platelet count, are normal. What is this patient’s diagnosis?
Chronic essential hypertension
Eclampsia
HELLP syndrome
Preeclampsia
None of them
A 71-year-old man comes to the emergency department with a 3-day history of malaise and
decreased urinary output. He denies any history of flank pain, hematuria, or prior voiding difficulty.
There is no fever, nausea, or vomiting, although his appetite is decreased. Past medical history is
significant for arthritis that is improved with ibuprofen. Recent illnesses include sinusitis for which
he was prescribed amoxicillin. He has been taking the amoxicillin for 3 days and has not noticed any
rash or other ill effects from the medication. His temperature is 36.7 C (98.0 F), blood pressure is
158/84 mm Hg, pulse is 98/min, and respirations are 18/min. He seems fatigued but not in distress.
His lungs are clear and his heart has a regular rhythm without murmurs. Abdomen is soft, nontender,
and without masses. He has no lesions of the penis and rectal examination reveals a smooth, small
prostate. Laboratory studies show a BUN of 54 mg/dL, creatinine of 3.5 mg/dL, and potassium of 6
mEq/L. Liver function studies are within normal limits. Urinalysis shows 2+ proteinuria, 11-50
WBC/hpf with casts, and 6-10 RBC/hpf. Urine osmolality is 250 mOsm/L. Which of the following
Acute
tubular necrosis
best describes
this patient’s condition?
Acute tubulointerstitial nephritis
C.
D.
E.
140.
A.
B. *
C.
D.
E.
141.
A.
B.
C.
D.
E. *
142.
A.
B. *
C.
D.
E.
143.
A.
B. *
C.
D.
E.
Postrenal acute renal failure
Prerenal acute renal failure
Unilateral renal obstruction
A 65 year old female is referred with a long history of hypertension and episodic urinary tract
infections. Dipstick analysis of the urine shows Blood +++ together with protein +++. Her Urea is 20
mmol/l (3-8) and Creatinine 280 micromol/l (60 – 100). An ultrasound of abdomen is requested and
shows left and right kidneys of 9 cm in size (10-12) without evidence of obstruction. Which one of
the following is the best investigation to diagnose the cause of her renal failure?
isotope renography
IV urography
renal angiography
renal biopsy
retrograde pyelography
In which of the following situations would a percutaneous needle biopsy of the kidney be most
helpful and appropriate?
Fever with suspected acute pyelonephritis
Premature neonate with suspected polycystic kidney disease
Prostatic hyperplasia with suspected hydronephrosis
Suspected renal cyst
Systemic lupus erythematosus and acute renal failure
A 31-year-old man was reviewed at a routine clinic visit having been undergoing continuous
ambulatory peritoneal dialysis for 6 months. He felt well, had no specific complaints, and was
clinically euvolaemic. He was using 4 . 1.5 L dextrose 1.36% exchanges. Investigations: serum
creatinine 899 .mol/L (60–110) urine volume 220 mL/day ultrafiltration volume 400 mL/day Kt/V
1.28 creatinine clearance 42 L/week What is the most appropriate management?
change nocturnal exchange to icodextrin
increase volume of exchanges to 2 L
no change to dialysis regimen
reduce to 3 . 1.5 L exchanges
start oral furosemide 500 mg daily
A 22-year-old man presented with a 2-week history of bilateral ankle swelling and mild ankle pain.
He had no past medical history and was taking no medication. He worked as a shop assistant, and
neither smoked nor used recreational drugs. On examination, his blood pressure was 110/55 mmHg,
and he had pitting oedema to mid- calf. His chest and abdomen were normal. He had small lymph
nodes palpable in his groins. Urinalysis showed protein 4+, blood 2+. Investigations: serum
sodium 141 mmol/L (137–144) serum potassium 4.1 mmol/L (3.5–4.9) serum creatinine 101
.mol/L (60–110) serum albumin 14 g/L (37–49) urine protein:creatinine ratio 762 mg/mmol (<15)
What investigation is likely to be most useful?
anti-neutrophil cytoplasmic antibody
antinuclear antibody
Epstein–Barr virus serology
protein electrophoresis
serum cryoglobulins
144.
A.
B.
C.
D.
E. *
145.
A.
B.
C.
D.
E. *
146.
A. *
B.
C.
D.
E.
147.
A. *
B.
C.
A 63-year-old woman presented with a 3-day history of haemoptysis. She also had a 4- month history
of lethargy and weight loss of 3 kg. On examination, she appeared pale and had bilateral red eyes,
but there were no other abnormalities. Urinalysis showed protein 2+, blood 3+. Investigations:
haemoglobin 89 g/L (115–165) white cell count 13.6 . 109/L (4.0–11.0) eosinophil count 0.8 .
109/L (0.04–0.40) platelet count 389 . 109/L (150–400) serum creatinine 389 .mol/L (60–110)
serum C-reactive protein 293 mg/L (<10) chest X-ray bilateral patchy shadowing in lower zones
What is the most likely diagnosis?
anti-glomerular basement membrane disease
Churg–Strauss syndrome
systemic lupus erythematosus
tubulointerstitial nephritis with uveitis
Wegener’s granulomatosis
A 62-year-old man was found to have proteinuria on routine testing. He had a 3-year history of
exertional angina but his symptoms had been well controlled since he had been taking atenolol 50
mg daily and amlodipine 10 mg daily. On examination, his blood pressure was 129/76 mmHg, his
jugular venous pressure was not raised, and he had mild ankle oedema, but his chest was clear.
Urinalysis showed protein 4+, blood 1+. Investigations: serum creatinine 92 .mol/L (60–110)
serum albumin 37 g/L (37–49) urinary protein:creatinine ratio 390 mg/mmol (<30) renal biopsy
histology membranous nephropathy What is the most appropriate treatment?
ciclosporin
cyclophosphamide and high-dose corticosteroids during alternate months for 6 months
furosemide
high-dose oral prednisolone
ramipril
A 53-year-old man presented to his general practitioner with a right inguinal hernia. He had a 6-year
history of hypertension that had been initially treated with atenolol but he had neither visited a
doctor nor taken any medication for 3 years. There was no other significant medical history. He
smoked 30 cigarettes per day. On examination, his blood pressure was 176/96 mmHg, his heart
sounds were normal and his chest was clear. The abdomen was normal. Fundoscopy revealed
bilateral dot haemorrhages, microaneurysms and hard exudates. Urinalysis showed protein 4+,
blood 2+. Investigations serum creatinine 176 .mol/L (60–110) fasting plasma glucose 16.7
mmol/L (3.0–6.0) urinary albumin:creatinine ratio 287 mg/mmol (<2.5) ultrasound scan of
kidneys normal appearances, left kidney 10.4 cm, right kidney 11.2 cm What is the most likely
diabetic
nephropathy
diagnosis?
focal and segmental glomerulosclerosis
hypertensive nephropathy
idiopathic membranous nephropathy
ischaemic nephropathy
A 52-year-old man presented with a 4-week history of nausea, anorexia, fever, bilateral flank pains
and polyuria. There was a past history of asthma and gastro-oesophageal reflux disease. His
medication comprised omeprazole and a compound, over-the-counter analgesic (paracetamol 500
mg/aspirin 300 mg/caffeine per tablet) for flank pain. On examination, his pulse was 72 beats per
minute and his blood pressure was 128/81 mmHg. There were no palpable abdominal masses.
Urinalysis showed blood trace. Investigations: haemoglobin 152 g/L (130–180) white cell count
8.6 . 109/L (4.0–11.0) platelet count 475 . 109/L (150–400) serum sodium 141 mmol/L
(137–144) serum potassium 5.2 mmol/L (3.5–4.9) serum urea 16.5 mmol/L (2.5–7.0) serum
creatinine 223 .mol/L (60–110) serum creatinine (5 months previously) 86 .mol/L (60–110) What is
acute
interstitial
nephritis
the most
likely diagnosis?
acute papillary necrosis
acute tubular necrosis
D.
E.
148.
A.
B.
C.
D.
E. *
149.
A.
B.
C. *
D.
E.
150.
A.
B.
C. *
D.
E.
Churg–Strauss syndrome
urinary tract obstruction
A 26-year-old woman with spina bifida had become increasingly confused over the preceding 12
hours. Her past medical history included recurrent urinary tract infections and anaphylaxis
secondary to penicillin. MRSA had been cultured from her urine during her previous two
admissions. On examination, her temperature was 38.9°C, her pulse was 112 beats per minute and
her blood pressure was 90/56 mmHg. Her Glasgow coma score was 10. Urinalysis showed
leucocytes and erythrocytes. Investigations: haemoglobin 136 g/L (115–165) white cell count 23.6
. 109/L (4.0–11.0) neutrophil count 18.4 . 109/L (1.5–7.0) serum sodium 132 mmol/L
(137–144) serum potassium 4.9 mmol/L (3.5–4.9) serum urea 14.7mmol/L (2.5–7.0) serum
creatinine 212 .mol/L (60–110) Which antibiotic should be included in her regimen?
cefuroxime
ciprofloxacin
piperacillin/tazobactam
rifampicin
vancomycin
A 74-year-old man presented with acute renal failure. He had a past medical history of
hypertension, ischaemic heart disease and type 2 diabetes mellitus. He smoked 25 cigarettes a day.
He had recently been found to be in atrial fibrillation and anticoagulation with warfarin was started.
Shortly before presentation he had developed lower abdominal pain, associated with watery
diarrhoea that had become blood-stained. On examination, he was afebrile, his pulse was 104 beats
per minute and irregularly irregular, his blood pressure was 176/94 mmHg, and he was euvolaemic.
He had abdominal tenderness below the umbilicus without guarding or rebound. No abdominal
masses were palpable. A purpuric rash was noted on his feet, legs and buttocks. His peripheral
pulses were absent below the knees bilaterally. Urinalysis showed blood 2+, protein 2+.
Investigations: haemoglobin 106 g/L (130–180) white cell count 22 . 109/L (4.0–11.0)
neutrophil count 18.0 . 109/L (1.5–7.0) lymphocyte count 3.1 . 109/L (1.5–4.0) eosinophil count
0.9 . 109/L (0.04–0.40) platelet count 164 . 109/L (150–400) erythrocyte sedimentation rate 75
mm/1st h (<20) serum creatinine 305 .mol/L (60–110) serum complement C3 55 mg/dL (65–190)
serum complement C4 12 mg/dL (15–50) What is the most likely cause of acute renal failure?
acute interstitial nephritis
aortic dissection
athero-embolic renal disease
cryoglobulinaemia
Henoch–Schonlein purpura
A 50-year-old woman was referred with a 6-month history of myalgia and arthralgia. She had a
history of recurrent renal stones and was undergoing intermittent lithotripsy. Her grandmother and
father had experienced renal problems. On examination, she had generalised muscle weakness but
her tendon reflexes, plantar responses and sensory examination were normal. Investigations: serum
sodium 140 mmol/L (137–144) serum potassium 2.9 mmol/L (3.5–4.9) serum chloride 118
mmol/L (95–107) serum bicarbonate 16 mmol/L (20–28) serum creatinine 185 .mol/L (60–110)
serum corrected calcium 2.05 mmol/L (2.20–2.60) 24-h urinary calcium 6.5 mmol (2.5–7.5)
urinary pH 7.0 What is the most likely diagnosis?
Bartter’s syndrome
cystinuria
distal renal tubular acidosis
hyporeninaemic hypoaldosteronism
proximal renal tubular acidosis
151.
A.
B.
C.
D.
E.
152.
A.
B.
C.
D. *
E.
153.
A.
B.
C. *
D.
E.
154.
A.
B. *
C.
D.
E.
A 28-year-old man was found to have protein 2+ on a routine urinalysis done during a life insurance
medical. His general practitioner confirmed this and referred him to the outpatient clinic. There had
been a similar finding at his occupational health screen when he started at university. He had
undergone further tests at the time and had been told there was nothing to worry about. Physical
examination was normal and his blood pressure was 118/76 mmHg. Investigations: serum urea 5.6
mmol/L (2.5–7.0) serum creatinine 92 .mol/L (60–110) estimated glomerular filtration rate
(MDRD) >60 mL/min (>60) urinalysis protein 2+ urinary protein:creatinine ratio (clinic sample)
103 mg/mmol (<30) urinary protein:creatinine ratio (early morning) 14 mg/mmol (<30) How should
he
be reassured
that he is at no increased risk of developing renal disease
thecan
patient
be advised?
he needs blood tests to exclude renal inflammation
he needs regular blood tests because he is at risk of worsening renal function
he should be started on an ACE inhibitor
he should have a renal biopsy to find out the cause of his proteinuria
A 54-year-old man presented after an episode of central chest pain lasting 60 minutes, which was
unrelieved by sublingual nitrate spray and required opioid analgesia. He had end-stage kidney
disease secondary to polycystic kidney disease and he had been dialysis-dependent for 3 years. On
examination, his pulse was 110 beats per minute and his blood pressure was 120/66 mmHg. He had
no signs of heart failure and no pericardial rub. Investigation: ECG ST segment depression in leads
V1 to V6 Which serum indicator is most specific for the diagnosis of acute coronary syndrome in
this
brainman?
natriuretic peptide
creatine kinase
creatine kinase MB fraction
troponin I
troponin T
A 78-year-old man presented with a 2-week history of ankle swelling and headache. He had a
4-year history of rheumatoid arthritis. His medication, which had remained unaltered for 3 years,
comprised methotrexate 10 mg weekly, folic acid 5 mg daily and diclofenac 75 mg daily. On
examination, his blood pressure was 188/122 mmHg and he had bilateral ankle oedema. There were
chronic changes of rheumatoid arthritis in the hands but no evidence of active synovitis.
Examination of the optic fundi showed grade 3 hypertensive retinopathy. Investigations: serum
creatinine 258 .mol/L (60–110) serum albumin 33 g/L (37–49) serum C-reactive protein 17 mg/L
(<10) 24-h urinary total protein 2.4 g (<0.2) What is the most likely renal diagnosis?
amyloidosis
analgesic nephropathy
hypertensive nephropathy
idiopathic membranous nephropathy
methotrexate nephrotoxicity
A 71-year-old man with IgA nephropathy was reviewed in the renal clinic. He complained of pain
in the right big toe of recent onset. His renal function was stable and he was otherwise well. He was
taking perindopril, amlodipine and thyroxine. On examination, he had swelling and erythema over
the distal joint of the toe. He was afebrile and did not look acutely ill. His body mass index was 32
kg/m2 (18–25). Investigations: serum creatinine 245 .mol/L (60–110) serum urate 0.68 mmol/L
(0.23–0.46) A clinical diagnosis of gout was made. What is the most appropriate treatment?
allopurinol
colchicine
diclofenac
prednisolone
probenecid
155.
A.
B.
C.
D. *
E.
156.
A.
B.
C.
D.
E. *
157.
A.
B.
C. *
D.
E.
158.
A.
B. *
C.
A 67-year-old woman presented with a 2-day history of increasing shortness of breath. She had
suffered a persistent cough for the previous 3 weeks and had lost 3 kg in weight. She had been
undergoing continuous ambulatory peritoneal dialysis (CAPD) for 3 months using 2.5 L exchanges.
She had a past medical history of diabetes mellitus, ischaemic heart disease and previous
tuberculosis. On examination, she was breathless on minimal exertion. Her blood pressure was
146/90 mmHg and her jugular venous pressure was visible at 4 cm above the sternal angle. There
was reduced air entry at the right base and this area was dull to percussion. She had mild ankle
oedema. Investigations: haemoglobin 110 g/L (115–165) white cell count 6.1 . 109/L (4.0–11.0)
platelet count 234 . 109/L (150–400) serum urea 24.0 mmol/L (2.5–7.0) serum creatinine 587
.mol/L (60–110) serum albumin 30 g/L (37–49) random plasma glucose 7.2 mmol/L pleural
fluid: total protein 20 g/L glucose 17 mmol/L lactate dehydrogenase 100 IU/L What is the most
cardiac
failure
likely
cause
of her shortness of breath?
fluid overload
mesothelioma
pleuroperitoneal leak
pulmonary tuberculosis
A 23-year-old woman presented with a 2-month history of lethargy. She had no other symptoms
and there was no other past medical history. Her mother confirmed that she had been healthy as a
child. On examination, her blood pressure was 178/110 mmHg and she had grade II hypertensive
retinopathy but there were no other abnormalities. Urinalysis showed protein 4+, blood 3+.
Investigations: serum creatinine 590 .mol/L (60–110) 24-h urinary total protein 3.9 g (<0.2)
ultrasound scan of kidneys right kidney 6 cm, irregular outline; left kidney 7 cm, irregular outline
What is the most likely diagnosis?
chronic glomerulonephritis
congenital renal dysplasia
fibromuscular dysplasia of the renal arteries
hypertensive nephropathy
reflux nephropathy
A 22-year-old woman attended for outpatient review and requested advice about family planning.
She had end-stage renal failure secondary to renal dysplasia, and had undergone pre-emptive
transplantation 6 months previously. She had gained 6 kg in weight since her transplant and had not
had any infections or episodes of rejection. Her current treatment comprised low-dose prednisolone,
and tacrolimus titrated against trough levels. Examination was normal. Her blood pressure was
142/78 mmHg. Investigations: serum urea 7.8 mmol/L (2.5–7.0) serum creatinine 116 .mol/L
(60–110) urinary protein:creatinine ratio 28 mg/mmol (<30) What is the most appropriate advice?
attempt to conceive without delay
avoid pregnancy because of risks to the fetus
delay conception for 6 months
introduce antihypertensive therapy before conception
substitute mycophenolate for tacrolimus before conception
An 87-year-old woman, who had been undergoing regular haemodialysis for 14 years, developed
infection in her arteriovenous fistula and infective endocarditis. The endocarditis was successfully
treated with 6 weeks of intravenous antibiotics but she then developed Clostridium difficile
diarrhoea. This persisted for over 6 weeks, during which time she had become malnourished and
required nasogastric tube feeding. She had also developed sacral and heel pressure sores. She asked
the nurses if she could stop dialysis but her family did not wish her to stop. What is the most
appropriate next action?
continue to dialyse her as long as the relatives wish it
explore further with her the reasons behind her decision
obtain a formal psychiatric assessment
D.
E.
159.
A.
B. *
C.
D.
E.
160.
A. *
B.
C.
D.
E.
161.
A.
B.
C. *
D.
E.
obtain legal advice
stop dialysis
A 47-year-old man, with end-stage renal failure secondary to polycystic kidney disease, underwent
successful renal transplantation. Routine immunosuppression included tacrolimus, mycophenolate
mofetil and prednisolone. At review 8 weeks later, he was feeling well apart from some lethargy
and a poor sleep pattern. On examination, he had a fine tremor of his hands and his blood pressure
was 126/80 mmHg. The renal graft was non-tender to palpation. Urinalysis revealed protein 2+.
Investigations: serum potassium 4.4 mmol/L (3.5–4.9) serum urea 5.9 mmol/L (2.5–7.0) serum
creatinine 104 .mol/L (60–110) serum corrected calcium 2.56 mmol/L (2.20–2.60) serum albumin
34 g/L (37–49) serum phosphate 0.56 mmol/L (0.8–1.4) serum alkaline phosphatase 187 U/L
(45–105) plasma parathyroid hormone 11.6 pmol/L (0.9–5.4) blood tacrolimus 11.2 .g/L (8–12)
What is the most likely cause of the hypophosphataemia?
Fanconi’s syndrome
hyperparathyroidism
malnutrition
mycophenolate mofetil
tacrolimus toxicity
A 64-year-old woman presented with a 4-week history of intermittent pain and numbness in her left
hand. She had noticed that her hand became pale at times, particularly in the cold. She had
long-standing type 2 diabetes mellitus and mild leg claudication, and had started haemodialysis via
a left brachial fistula 2 months previously. The symptoms in her left hand were worse during
dialysis. On examination, the left hand was paler than the right but all pulses were present and
equal. The brachial fistula was working well. Pain and fine touch sensation were reduced in the
thumb and first two fingers of the left hand. What is the most likely diagnosis?
carpal tunnel syndrome
cervical spondylosis
diabetic neuropathy
reflex sympathetic dystrophy
Steal syndrome
A 50-year-old woman presented to her general practitioner with a 3-week history of malaise and
oliguria. On examination, her blood pressure was 150/98 mmHg, her jugular venous pressure was
elevated to 6 cm and she had a soft pericardial friction rub audible over the precordium. She had
bilateral pitting oedema to the knees. Urinalysis showed blood 2+, protein 2+. Investigations:
serum sodium 136 mmol/L (137–144) serum potassium 5.7 mmol/L (3.5–4.9) serum urea 24.0
mmol/L (2.5–7.0) serum creatinine 779 .mol/L (60–110) serum complement C3 46 mg/dL
(65–190) serum complement C4 20 mg/dL (15–50) serum immunoglobulin G 6.8 g/L (6.0–13.0)
serum immunoglobulin A 4.3 g/L (0.8–3.0) serum immunoglobulin M 1.2 g/L (0.4–2.5)
anti-double-stranded DNA antibodies (ELISA) 35 U/mL (<73) anti-glomerular basement membrane
antibodies negative anti-neutrophil cytoplasmic antibodies negative What is the most likely
focal segmental glomerulosclerosis
diagnosis?
IgA nephropathy
infective endocarditis
membranous glomerulonephritis
renal limited vasculitis
162.
A.
B.
C.
D.
E. *
163.
A.
B.
C.
D.
E. *
164.
A. *
B.
C.
D.
E.
165.
A.
B.
C.
D. *
E.
A 65-year-old woman with a 30-year history of rheumatoid arthritis presented with a 6- week history
of progressive ankle oedema. She had been treated with regular gold injections for 5 years, but these
had been stopped 9 months previously. She had also been taking diclofenac for the past 2 years. On
examination, she had pitting oedema to her knees and a sacral pad. Investigations: haemoglobin 106
g/L (115–165) platelet count 164 . 109/L (150–400) serum sodium 143 mmol/L (137–144)
serum potassium 4.4 mmol/L (3.5–4.9) serum creatinine 223 .mol/L (60–110) serum albumin 19
g/L (37–49) 24-h urinary total protein 7.8 g (<0.2) What is the most likely diagnosis?
crescentic glomerulonephritis
gold-induced membranous nephropathy
interstitial nephritis
minimal change nephropathy
renal amyloid
A 78-year-old woman presented with shortness of breath and oedema. She had felt generally unwell
for the last few weeks with increasing tiredness, loss of appetite and abdominal discomfort. Her
serum creatinine was 105 .mol/L (60–110). She was treated with intravenous furosemide at doses
up to 120 mg twice a day, but there was no improvement in her symptoms and her renal function
deteriorated. On examination, her blood pressure was 125/65 mmHg and her jugular venous pressure
was not elevated. She had oedema to the thigh. Auscultation of the chest revealed dullness to
percussion on the left. Investigations: serum sodium 128 mmol/L (137–144) serum potassium 3.2
mmol/L (3.5–4.9) serum urea 31.5 mmol/L (2.5–7.0) serum creatinine 351 .mol/L (60–110)
serum albumin 15 g/L (37–49) 24-h urinary total protein 15.8 g (<0.2) What is the most appropriate
increase
dose
of furosemide
next
step in
management?
intravenous sodium chloride 0.9%
start enalapril
start metolazone
ultrafiltration
A 60-year-old woman developed atrial fibrillation and was treated with warfarin. Four weeks later,
she presented with painful discolouration of lower limb extremities and acute renal failure. Her
renal function had previously been normal. What is the most likely diagnosis?
embolisation
cryoglobulinaemia
Henoch–Schonlein purpura
IgA nephropathy
systemic small vessel vasculitis
A 55-year-old man with end-stage renal disease, who had been undergoing haemodialysis for 3
years, complained of generalised muscle stiffness and pain in his lower back. He had lost 4 kg in
weight over the previous 4 months. Clinical examination was unremarkable. Investigations: serum
urea 32 mmol/L (2.5–7.0) serum creatinine 1208 .mol/L (60–110) serum corrected calcium 2.30
mmol/L (2.20–2.60) serum phosphate 3.1 mmol/L (0.8–1.4) serum total protein 75 g/L (61–76)
serum albumin 39 g/L (37–49) serum alkaline phosphatase 167 U/L (45–105) serum aluminium 3
.g/L (<10) plasma parathyroid hormone 36 pmol/L (0.9–5.4) What is the most likely bone
decreased bone mineralisation
abnormality?
decreased osteoblast activity
disrupted continuity of the trabeculae
increased osteoclast activity
thin osteoid seams
166.
A.
B.
C. *
D.
E.
167.
A.
B.
C.
D. *
E.
168.
A.
B.
C.
D. *
E.
169.
A.
B.
C.
D. *
E.
A 26-year-old woman presented with a facial rash and arthralgia. Her blood pressure was 116/66
mmHg. Urinalysis showed blood 2+, protein 2+. Investigations: serum creatinine 88 .mol/L
(60–110) 24-h urinary total protein 0.8 g (<0.2) anti-double-stranded DNA antibodies (ELISA)
229 U/mL (<73) A diagnosis of systemic lupus erythematosus was made and she was treated with
prednisolone 60 mg daily and azathioprine 2 mg/kg/day. On the same day, a renal biopsy was
performed that showed class II lupus nephritis. What is the most appropriate further management?
add ciclosporin
add intravenous methylprednisolone 1 g for 3 days
no change
stop azathioprine, start cyclophosphamide
stop azathioprine, start mycophenolate mofetil
A 64-year-old man undergoing maintenance haemodialysis was reviewed. He had suffered a
myocardial infarction 9 months earlier. His medication comprised alfacalcidol 0.25 micrograms
once daily, calcium carbonate 500 mg three times daily, ramipril 5 mg once daily, simvastatin 10
mg at night and subcutaneous epoetin beta 3000 units twice weekly. Investigations: haemoglobin 96
g/L (130–180) white cell count 6.7* 109/L (4.0–11.0) platelet count 175*109/L (150–400) serum
ferritin 185 .g/L (15–300) What is the most appropriate next management step?
blood transfusion
change epoetin beta to darbepoetin
increase dose of epoetin beta
intravenous iron
stop ramipril
A 37-year-old woman presented with acute renal failure and pulmonary haemorrhage resulting from
anti-neutrophil cytoplasmic antibody-positive vasculitis, and was treated with immunosuppression,
dialysis and plasma exchange. Three months later, the vasculitis was not clinically active and she
had recovered sufficient renal function to cease dialysis. What is the most likely mode of action of
plasma exchange in this patient?
anti-inflammatory effects of replacement plasma
removal of activated complement
removal of inflammatory cytokines
removal of pathogenic antibodies
replenishment of normal immunoglobulins
A 37-year-old woman with a 4-year history of Raynaud’s phenomenon, arthralgia, weight loss,
muscle tenderness and intermittent malaise was referred by her general practitioner. On examination,
she had indurated thickening of the skin distal to the metacarpophalangeal joints. There was no
active synovitis, rash or nail abnormalities. Her blood pressure was 158/94 mmHg. She weighed 48
kg. Investigations: haemoglobin 110 g/L (115–165) white cell count 4.0 . 109/L (4.0–11.0)
platelet count 135 . 109/L (150–400) erythrocyte sedimentation rate 55 mm/1st h (<20) serum
urea 7.0 mmol/L (2.5–7.0) serum creatinine 144 .mol/L (60–110) serum creatine kinase 210 U/L
(24–170) urinary protein:creatinine ratio 105 mg/mmol (<30) serum complement C3 74 mg/dL
(65–190) serum complement C4 22 mg/dL (15–50) serum C-reactive protein 42 mg/L (<10)
antinuclear antibodies positive at 1:128 dilution anti-La antibodies negative anti-RNP antibodies
negative anti-Scl-70 antibodies negative anti-Ro antibodies positive anti-Sm antibodies negative
dermatomyositis
What is the most likely diagnosis?
Sjogren's disease
systemic lupus erythematosus
systemic sclerosis
undifferentiated connective tissue disease
170.
A. *
B.
C.
D.
E.
171.
A.
B. *
C.
D.
E.
172.
A.
B.
C.
D. *
E.
173.
A. *
B.
C.
D.
E.
A 50-year-old man who was undergoing maintenance haemodialysis was reviewed. His medication
comprised alfacalcidol 0.25 micrograms daily, calcium carbonate 500 mg three times daily, ramipril
5 mg daily and simvastatin 10 mg daily. The dialysate calcium concentration was 1.25 mmol/L.
Investigations: serum corrected calcium 2.60 mmol/L (2.20–2.60) serum phosphate 1.67 mmol/L
(0.8–1.4) plasma parathyroid hormone 28.0 pmol/L (0.9–5.4) What is the most appropriate next
management step?
change calcium carbonate to sevelamer
dialyse against lower calcium dialysate
no change
omit alfacalcidol
prescribe cinacalcet
A 22-year-old woman was admitted by the obstetric service in the 30th week of her first pregnancy.
She had been treated with antibiotics for dysuria at 12 weeks. On admission, she complained of
dysuria and suprapubic discomfort. General examination was normal. Her temperature was 36.8°C,
her pulse was 78 beats per minute and her blood pressure was 104/62 mmHg. There was no
peripheral oedema and fundal height was appropriate. Investigations: haemoglobin 118 g/L
(115–165) white cell count 7.6 . 109/L (4.0–11.0) serum urea 3.0 mmol/L (2.5–7.0) serum
creatinine 46 .mol/L (60–110) serum C-reactive protein 8 mg/L (<10) midstream urine
microscopy: white cells >50/.L (<10) culture >105 E. coli per mL, sensitive to amoxicillin,
ciprofloxacin, trimethoprim ultrasound scan of renal tract minimal right hydronephrosis What
DTPA
isotopeisrenography
investigation
most appropriate?
monthly midstream urine cultures
MR urography
retrograde pyelography
urodynamic study
A 70-year-old man with hypertension was admitted with a 7-day history of worsening confusion.
His wife reported a 13-kg weight loss, anorexia and vomiting after food over a 6-month period. On
examination, he had reduced skin turgor. His blood pressure was 120/75 mmHg and his respiratory
rate was 12 breaths per minute. His Glasgow coma score was 12. Investigations: serum sodium 128
mmol/L (137–144) serum potassium 2.5 mmol/L (3.5–4.9) serum chloride 75 mmol/L (95–107)
serum bicarbonate 48 mmol/L (20–28) serum urea 34.0 mmol/L (2.5–7.0) serum creatinine 200
.mol/L (60–110) plasma osmolality 300 mosmol/kg (278–300) urinary pH 5.8 urinary sodium 5
mmol/L (<10) urinary osmolality 650 mosmol/kg (350–1000) serum cortisol (09.00 h) 650 nmol/L
(200–700) What is the most likely diagnosis?
Addisonian crisis
Bartter’s syndrome
distal renal tubular acidosis
pyloric stenosis
syndrome of inappropriate antidiuretic hormone secretion (SIADH)
A 78-year-old man presented with a 3-month history of malaise. On examination, his blood pressure
was 135/72 mmHg. Investigations: haemoglobin 100 g/L (130–180) serum sodium 133 mmol/L
(137–144) serum potassium 6.4 mmol/L (3.5–4.9) serum creatinine 608 .mol/L (60–110)
ultrasound scan of kidneys grossly hydronephrotic right kidney with thin cortex; left kidney with
moderate hydronephrosis but otherwise normal appearance What is the most appropriate next step
in management?
percutaneous nephrostomy of the left kidney
percutaneous nephrostomy of the right kidney
retrograde ureteric stent on the left
retrograde ureteric stent on the right
urgent dialysis
174.
A.
B.
C.
D.
E. *
175.
A.
B.
C.
D.
E. *
176.
A.
B.
C.
D.
E. *
177.
A.
B.
C.
D. *
E.
178.
A.
B.
C. *
D.
A 55-year-old man presented with left ureteric colic. He later passed a stone and analysis showed it
to be composed of calcium oxalate. He had experienced no previous symptomatic nephrolithiasis
and had no other past medical history. Investigations: serum corrected calcium 2.35 mmol/L
(2.20–2.60) serum urate 0.40 mmol/L (0.23–0.46) 24-h urinary calcium 8.5 mmol (2.5–7.5) 24-h
urinary urate 1.6 mmol (<3.6) 24-h urinary oxalate 0.3 mmol (0.14–0.46) What is the most
appropriate way to prevent further stone formation?
alkalinisation of the urine
restriction of dietary calcium
restriction of dietary oxalate
restriction of dietary protein
restriction of dietary sodium
A 45-year-old man on regular haemodialysis complained of weakness and exertional fatigue. On
examination, his blood pressure was 170/105 mmHg (pre-dialysis) and 160/95 mmHg (post-dialysis).
Investigations pre-dialysis revealed: Haemoglobin9.0g/dl; serum potassium6.9; serum
creatinine1250; serum corrected calcium2.1 mmol/l. Which intervention is most likely to improve his
symptoms:
increase
haemoglobin with epoetin
increase the length of each dialysis session
lower the potassium in the dialysate
improve blood pressure control with ramipril
all of them
A 42 year old female is admitted following an overdose of Diazepam and alcohol. On examination
she was unconscious with a core temperature of 34.5°C and a blood pressure of 110/80 mmHg.
Investigations reveal: Creatinine242 micromol/L (60-100); AST500 U/L (0-40); Gamma GT35 U/L
(<50); urine microscopy no cells or organisms ; urine dipstick analysis blood+++ ; Ultrasound
abdomen normal. Which one of the following is the most likely cause of these findings?
Associated Paracetamol poisoning
Chronic renal failure
Dehydration
Hypothermia
Rhabdomyolysis
A 70 year old man underwent emergency surgery for an acute abdomen. Following surgery she was
noted to have become oliguric. Investigations revealed the following: sodium121 mmol/L (137-144);
potassium6.6 mmol/L (3.5-4.9); chloride92 mmol/L (95-107); Urea17.2 mmol/l (3-8); creatinine250
micromol/L (60-110); Arterial blood gas pH 7.16; Standard Bicarbonate15.6 (21-27). What is the
calculated anion gap for this patient?
5 mmol/L
10 mmol/L
15 mmol/L
20 mmol/L
25 mmol/L
A 68 year old male diagnosed with nephrotic syndrome receives steroid therapy without benefit. His
investigations show an albumin of 20 g/L (37 - 49), Total cholesterol of 12 mmol/l, dipstick
urinanalysis reveals +++ protein and a renal biopsy shows focal segmental glomerulosclerosis.
Which one of the following is most likely to preserve renal function?
dietary salt restriction
low dietary protein intake
ramipril
simvastatin
E.
179.
A.
B.
C.
D. *
E.
180.
A. *
B.
C.
D.
E.
181.
A. *
B.
C.
D.
E.
182.
A. *
B.
C.
D.
E.
183.
A. *
B.
C.
D.
warfarin
50-A 25 year old woman who is 20 weeks pregnant is diagnosed with pyelonephritis. She had
suffered recurrent urinary infections since childhood and her family history reveals that her mother
had a history of hypertension and had been told she had a kidney problem. Examination was normal
and Urea and Creatinine were both normal What is the most likely diagnosis?
autosomal dominant polycystic kidney disease
bladder outlet obstruction
normal physiological urinary stasis of pregnancy
reflux nephropathy
renal stone disease
A 14 year old boy is admitted following an epileptic seizure. This is his first seizure and on
examination you note learning difficulties, a bumpy rash on his nose and cheeks and a blood pressure
of 135/82 mmHg. His blood results are normal but a urine dipstick reveals trace levels of blood and
an ultrasound shows cysts in both kidneys. What is the diagnosis?
Tuberous sclerosis
Polycystic kidney disease
Glioblastoma multiforme
MEN1
Neurofibromatosis
A 3 year old is brought to the GP as she is thought to be drinking and urinating excessively by her
parents. She has recently began wetting the bed again after being potty trained last year. She appears
very small for her age. Bloods reveal renal impairment, urinalysis normal and an ultrasound is
organised. On ultrasound the kidneys appear small and there is evidence of multiple small medullary
cysts. What is the most likely diagnosis?
Nephronophthisis
Medullary sponge kidney
Posterior urethral valve
Chronic pyelonephritis
Polycystic kidney disease
A 72 year old gentleman with type 2 diabetes mellitus has presented with pyrexia, flank pain and
haematuria. Bloods reveal an acute deterioration of his renal function. Urine dipstick reveals blood,
protein and low urine specific gravity. An ultrasound suggests hydronephrosis. A CT scan is
performed and reveals evidence of renal pelvic filling defects, ring shadows and irregular papillae.
What is the most likely diagnosis?
Papillary necrosis
Pyelonephritis
Renal stones
Hydronephrosis
Glomerulonephritis
A 45 year old female is admitted with cellulitus requiring IV antibiotics. She is commenced on IV
benzylpenicillin and IV flucloxacillin. She is commenced on Diclofenac for pain. After 3 days she
she develops pyrexia, arthralgia, maculopapular rash, haematuria and oliguria. Bloods reveal acute
renal failure and eosinophilia. A renal biopsy reveals interstitial cellular infiltrate with eosinophils
present and variable tubular necrosis. What is the most likely diagnosis?
Penicillin induced acute tubulointerstitial nephritis
NSAID induced acute tubulointerstitial nephritis
Chronic tubulointerstitial nephritis
Glomerulonephritis
E.
184.
A.
B. *
C.
D.
E.
185.
A. *
B.
C.
D.
E.
186.
A. *
B.
C.
D.
E.
187.
A. *
B.
C.
D.
E.
188.
A. *
B.
C.
D.
Anaphylaxis
70 year old male with previous radiation therapy for colorectal carcinoma presents with non specific
symptoms of weight loss, malaise, nausea and dull abdominal and back pain. He has also noticed
increased frequency and hesitancy of urination. Bloods reveal a normochromic anaemia, increased
ESR and renal failure.. His renal failure does not improve with fluids and an ultrasound is organised.
This reveals a retroperitoneal mass. What is the most likely diagnosis?
Amyloidosis
Retroperitoneal fibrosis
Aortic aneurysm
Pancreatic carcinoma
Recurrence of colorectal carcinoma
A patient is admitted with decompensated liver disease and tense ascites. His bloods reveal an acute
renal failure. He is not on any nephrotoxic drugs. He had been previously on spironolactone and
furosemide for prophylaxis of ascites however these were stopped some months ago. His blood
pressure is 130/80 mmHg and his renal failure does not respond to fluids. A urinalysis shows a trace
amount of protein but nil else. An ultrasound of the renal tract shows no evidence of obstruction.
What is the most likely diagnosis?
Hepatorenal syndrome
Hypovolaemia
Glomerulonephritis
Previous furosemide use
Sepsis
A 75 year old gentleman presents with difficulty voiding urine. He has a past medical history of Type
2 Diabetes Mellitus which is well controlled. For the last few months he has notice he has been
suffering from frequency, nocturia and hesitancy when urinating. He now presents as he has been
unable to pass urine for the last 24 hours and is complaining of suprapubic discomfort. For the last
few days he states he has been feeling unwell with suprapubic discomfort and dysuria. On
examination his bladder is palpable. His heart rate is 100 and blood pressure is 150/90.
Obstructive Uropathy
Diabetic Nephropathy
Sepsis
Hypovolaemia
Hypertension
A 65 year old female with multiple myeloma presents with malaise. She is found to be in acute renal
failure and is hypercalcaemic. She has been taking paracetamol for pain but nil else. A urinalysis
reveals a trace of blood and protein. Which of the following is the first action you should take?
Commence IV fluids
Organise Urgent Renal Ultrasound
IV Pamidronate
Cyclophosphamide
Dialysis
A 60 year old male is noted to have macroglossia, hepatomegaly and has waxy papules in the eyelids,
neck and groin. He complains of symptoms of carpal tunnel syndrome. He has now developed
oedema and is found to have nephrotic syndrome. What is the most likely diagnosis?
Amyloidosis
Glomerulonephritis
Minimal change disease
Scleroderma
E.
189.
A.
B.
C.
D.
E. *
190.
A. *
B.
C.
D.
E.
191.
A. *
B.
C.
D.
E.
192.
A. *
B.
C.
D.
E.
Hypercholesterolaemia
A 61 year old female presents with haematuria. She has been on sulphasalazine for many years for
rheumatoid arthritis and takes ramipril for hypertension. Her blood pressure is 140/82 and she has
classical rheumatoid features on her hands. Her urine is positive for blood and protein, her
haemoglobin is 11 and creatinine levels are slightly raised. What investigation should be performed
next?
Cystoscopy
Urine culture
Renal ultrasound
CT KUB
Renal biopsy
A 73 year old gentleman presents to his GP with significant peripheral oedema. He has a past
medical history of well controlled type 2 diabetes mellitus. He is also complaining of fatigue. Over
the last few months he has been taking diclofenac for his osteoarthritis. A urinalysis leaves protein 3+
and bloods reveal a urea of 12.1 and a creatinine of 210. These had been normal 3 months ago.
Which of the following would you expect to find on renal biopsy?
Tubulointerstitial Nephritis
Membranous Nephropathy
Minimal Change Disease
Diabetic Nephropathy
Nil
A 37 year old male has a history of rhinorrhoea , recurrent sinusitis, joints pains and cough with
occasional haemoptysis. On dipstick of his urine there is evidence of proteinuria and haematuria.
Bloods reveal acute renal failure and autoantibodies reveal a positive cANCA. What is the most
likely diagnosis?
Wegeners Granulomatosis
Churg Strauss syndrome
Haemolytic Uraemic syndrome
Microscopic polyangitis
IgA nephropathy
A 56 year old male with known peripheral vascular disease and ischaemic heart disease is currently
on amlodipine and bendroflumethiazide for blood pressure control. On his last few blood pressure
checks he has been found to be hypertensive. He is therefore commenced on ramipril. He had his
bloods checked one week later and was found to be in acute renal failure. Given the most likely
diagnosis, what is the most appropriate management?
Stop ACE inhbitor, Aspirin, Statin and optimal blood pressure control
Renal angioplasty
Stop ACE inhbitor until bloods normalise then restart at lower dose
Ultrasound
Renal artery thrombolysis